How To Present Algebraic Topology To Non-Mathematicians?












38












$begingroup$


I am writing my master thesis in algebraic topology (fundamental groups), and, as a system in my school, students must write one page about their theses explaining for non-mathematicians the purpose of the study (in general) and why it is important in the real life. I don't know actually how shall I start, because I'm not allowed to use some mathematical terminologies like "Topological Space", for instance. My teacher told me it's important to convince the "politicians" to support financially the research in the department of mathematics mainly in algebraic topology to show them that we are doing something useful. Furthermore; it is to make the new students like this kind of mathematics.



So any suggestion or idea will be very appreciated!










share|cite|improve this question











$endgroup$








  • 52




    $begingroup$
    Good luck. $,,$
    $endgroup$
    – Tyler
    Oct 25 '13 at 20:24








  • 28




    $begingroup$
    Algebraic topology isn't important in real life, so you face an impossible task.
    $endgroup$
    – Potato
    Oct 25 '13 at 20:35






  • 3




    $begingroup$
    You should probably focus less on "this is what I do" and more on "this is what it's good for".
    $endgroup$
    – Hurkyl
    Oct 25 '13 at 20:41






  • 5




    $begingroup$
    I heard an example of a researcher who put in an NSF proposal abstract the standard example of a coffee cup being equivalent to a doughnut. Then some legislator read the abstract and started railing about how the government is funding ridiculous research projects. Sometimes it's good to be somewhat obfuscatory. ;)
    $endgroup$
    – Cheerful Parsnip
    Oct 25 '13 at 23:47






  • 12




    $begingroup$
    @Potato The Institute for Mathematics and its Applications will be devoting the next year to proving you wrong :) ima.umn.edu/2013-2014
    $endgroup$
    – Gyu Eun Lee
    Oct 26 '13 at 0:18


















38












$begingroup$


I am writing my master thesis in algebraic topology (fundamental groups), and, as a system in my school, students must write one page about their theses explaining for non-mathematicians the purpose of the study (in general) and why it is important in the real life. I don't know actually how shall I start, because I'm not allowed to use some mathematical terminologies like "Topological Space", for instance. My teacher told me it's important to convince the "politicians" to support financially the research in the department of mathematics mainly in algebraic topology to show them that we are doing something useful. Furthermore; it is to make the new students like this kind of mathematics.



So any suggestion or idea will be very appreciated!










share|cite|improve this question











$endgroup$








  • 52




    $begingroup$
    Good luck. $,,$
    $endgroup$
    – Tyler
    Oct 25 '13 at 20:24








  • 28




    $begingroup$
    Algebraic topology isn't important in real life, so you face an impossible task.
    $endgroup$
    – Potato
    Oct 25 '13 at 20:35






  • 3




    $begingroup$
    You should probably focus less on "this is what I do" and more on "this is what it's good for".
    $endgroup$
    – Hurkyl
    Oct 25 '13 at 20:41






  • 5




    $begingroup$
    I heard an example of a researcher who put in an NSF proposal abstract the standard example of a coffee cup being equivalent to a doughnut. Then some legislator read the abstract and started railing about how the government is funding ridiculous research projects. Sometimes it's good to be somewhat obfuscatory. ;)
    $endgroup$
    – Cheerful Parsnip
    Oct 25 '13 at 23:47






  • 12




    $begingroup$
    @Potato The Institute for Mathematics and its Applications will be devoting the next year to proving you wrong :) ima.umn.edu/2013-2014
    $endgroup$
    – Gyu Eun Lee
    Oct 26 '13 at 0:18
















38












38








38


19



$begingroup$


I am writing my master thesis in algebraic topology (fundamental groups), and, as a system in my school, students must write one page about their theses explaining for non-mathematicians the purpose of the study (in general) and why it is important in the real life. I don't know actually how shall I start, because I'm not allowed to use some mathematical terminologies like "Topological Space", for instance. My teacher told me it's important to convince the "politicians" to support financially the research in the department of mathematics mainly in algebraic topology to show them that we are doing something useful. Furthermore; it is to make the new students like this kind of mathematics.



So any suggestion or idea will be very appreciated!










share|cite|improve this question











$endgroup$




I am writing my master thesis in algebraic topology (fundamental groups), and, as a system in my school, students must write one page about their theses explaining for non-mathematicians the purpose of the study (in general) and why it is important in the real life. I don't know actually how shall I start, because I'm not allowed to use some mathematical terminologies like "Topological Space", for instance. My teacher told me it's important to convince the "politicians" to support financially the research in the department of mathematics mainly in algebraic topology to show them that we are doing something useful. Furthermore; it is to make the new students like this kind of mathematics.



So any suggestion or idea will be very appreciated!







algebraic-topology soft-question education big-list






share|cite|improve this question















share|cite|improve this question













share|cite|improve this question




share|cite|improve this question








edited Oct 10 '16 at 19:26









dgwade

52




52










asked Oct 25 '13 at 20:23









RahmanRahman

606519




606519








  • 52




    $begingroup$
    Good luck. $,,$
    $endgroup$
    – Tyler
    Oct 25 '13 at 20:24








  • 28




    $begingroup$
    Algebraic topology isn't important in real life, so you face an impossible task.
    $endgroup$
    – Potato
    Oct 25 '13 at 20:35






  • 3




    $begingroup$
    You should probably focus less on "this is what I do" and more on "this is what it's good for".
    $endgroup$
    – Hurkyl
    Oct 25 '13 at 20:41






  • 5




    $begingroup$
    I heard an example of a researcher who put in an NSF proposal abstract the standard example of a coffee cup being equivalent to a doughnut. Then some legislator read the abstract and started railing about how the government is funding ridiculous research projects. Sometimes it's good to be somewhat obfuscatory. ;)
    $endgroup$
    – Cheerful Parsnip
    Oct 25 '13 at 23:47






  • 12




    $begingroup$
    @Potato The Institute for Mathematics and its Applications will be devoting the next year to proving you wrong :) ima.umn.edu/2013-2014
    $endgroup$
    – Gyu Eun Lee
    Oct 26 '13 at 0:18
















  • 52




    $begingroup$
    Good luck. $,,$
    $endgroup$
    – Tyler
    Oct 25 '13 at 20:24








  • 28




    $begingroup$
    Algebraic topology isn't important in real life, so you face an impossible task.
    $endgroup$
    – Potato
    Oct 25 '13 at 20:35






  • 3




    $begingroup$
    You should probably focus less on "this is what I do" and more on "this is what it's good for".
    $endgroup$
    – Hurkyl
    Oct 25 '13 at 20:41






  • 5




    $begingroup$
    I heard an example of a researcher who put in an NSF proposal abstract the standard example of a coffee cup being equivalent to a doughnut. Then some legislator read the abstract and started railing about how the government is funding ridiculous research projects. Sometimes it's good to be somewhat obfuscatory. ;)
    $endgroup$
    – Cheerful Parsnip
    Oct 25 '13 at 23:47






  • 12




    $begingroup$
    @Potato The Institute for Mathematics and its Applications will be devoting the next year to proving you wrong :) ima.umn.edu/2013-2014
    $endgroup$
    – Gyu Eun Lee
    Oct 26 '13 at 0:18










52




52




$begingroup$
Good luck. $,,$
$endgroup$
– Tyler
Oct 25 '13 at 20:24






$begingroup$
Good luck. $,,$
$endgroup$
– Tyler
Oct 25 '13 at 20:24






28




28




$begingroup$
Algebraic topology isn't important in real life, so you face an impossible task.
$endgroup$
– Potato
Oct 25 '13 at 20:35




$begingroup$
Algebraic topology isn't important in real life, so you face an impossible task.
$endgroup$
– Potato
Oct 25 '13 at 20:35




3




3




$begingroup$
You should probably focus less on "this is what I do" and more on "this is what it's good for".
$endgroup$
– Hurkyl
Oct 25 '13 at 20:41




$begingroup$
You should probably focus less on "this is what I do" and more on "this is what it's good for".
$endgroup$
– Hurkyl
Oct 25 '13 at 20:41




5




5




$begingroup$
I heard an example of a researcher who put in an NSF proposal abstract the standard example of a coffee cup being equivalent to a doughnut. Then some legislator read the abstract and started railing about how the government is funding ridiculous research projects. Sometimes it's good to be somewhat obfuscatory. ;)
$endgroup$
– Cheerful Parsnip
Oct 25 '13 at 23:47




$begingroup$
I heard an example of a researcher who put in an NSF proposal abstract the standard example of a coffee cup being equivalent to a doughnut. Then some legislator read the abstract and started railing about how the government is funding ridiculous research projects. Sometimes it's good to be somewhat obfuscatory. ;)
$endgroup$
– Cheerful Parsnip
Oct 25 '13 at 23:47




12




12




$begingroup$
@Potato The Institute for Mathematics and its Applications will be devoting the next year to proving you wrong :) ima.umn.edu/2013-2014
$endgroup$
– Gyu Eun Lee
Oct 26 '13 at 0:18






$begingroup$
@Potato The Institute for Mathematics and its Applications will be devoting the next year to proving you wrong :) ima.umn.edu/2013-2014
$endgroup$
– Gyu Eun Lee
Oct 26 '13 at 0:18












13 Answers
13






active

oldest

votes


















28












$begingroup$

Here are two examples of algebraic topology (actually homology theory) being applied to "real world" problems.



1) Ghrist's paper "Homological sensor networks." (pdf)



2) Topological data analysis, as pioneered by Gunnar Carlsson.






share|cite|improve this answer











$endgroup$





















    20












    $begingroup$

    Explain that in a society that values learning, the pursuit of abstract knowledge is an end unto itself. This is how pure mathematicians have answered that question for millennia and is, as far as I know, the only truly irrefutable answer.






    share|cite|improve this answer









    $endgroup$









    • 14




      $begingroup$
      Not very helpful. Sure abstract knowledge is worth pursuing, but you're generally a bit more selective about what abstract knowledge you pursue. If I made up a meaningless formal system of axioms right now, and then acted condescending towards people who didn't find them interesting, I'd be rightly considered mad. What aspects of algebraic topology make its abstract study interesting?
      $endgroup$
      – Jack M
      Oct 26 '13 at 0:18






    • 3




      $begingroup$
      The OP specifically mentioned reasons for explaining algebraic topology to laymen: to get financial support for his work, and to motivate students. I don't think this answer does a good job at achieving either goal. This may be an irrefutable answer, but it's not a useful one.
      $endgroup$
      – Gyu Eun Lee
      Oct 26 '13 at 4:31






    • 9




      $begingroup$
      I think there are some really cool arguments to be had here. One of my favorite recent articles is called "Understanding is a Poor Substitute for Convexity" and it argues that science is productive simply because it is able to create convex payoff functions, and that supporting research at arbitrarily and at random is already quite a good strategy, and maybe even near-optimal. Together with the philosophical sentiment that we should be a society that values learning—well, it's compelling. I love it. But it is most certainly not a way to get any particular project funded.
      $endgroup$
      – Slade
      Oct 26 '13 at 8:48






    • 2




      $begingroup$
      This answer only tells me that you're doing something for which you don't see any practical application. It doesn't tell me what you're doing, and it doesn't tell me that your pursuit has any kind of long-term goal or structure. Are you doing anything other than intellectual masturbation all day?
      $endgroup$
      – Jonas
      Oct 26 '13 at 13:14






    • 1




      $begingroup$
      @Jonas That argument serves equally well to defund scholars in the philosophy department, the classics department, the English department, the art history department, and even the astronomy department. Its wide applicability betrays its emptiness. Yes, scholarship can seem like intellectual masturbation to outsiders, but that observation is neither recent nor intelligent.
      $endgroup$
      – TBrendle
      Oct 28 '13 at 3:39





















    15












    $begingroup$

    It may not be "useful", but you can explain the subject to a nontechnical audience by relating it to "wire and string" puzzles. The solvability of wire-and-string puzzles can basically be reduced to questions about the fundamental group of the complementary ambient space surrounding the metal wire.



    For example, look at the following "astroknot puzzle", where the goal is to remove the long flexible string from the solid metal,
    enter image description here



    This also, incidentally, leads to one of the best ways to solve such puzzles as a human, which is to imagine that the inflexible metal can bend into a form where the puzzle is easy, and then imagine how the nearby air would deform under the reverse transformation.






    share|cite|improve this answer











    $endgroup$









    • 2




      $begingroup$
      I don't think you're being totally genuine by saying the solvability of such a puzzle is only determined by the topology of the ambient complement. In such puzzles, the lengths of both the strings, and the shapes/lengths of the rigid parts can affect whether the puzzle is solvable. It is true though that the topology of the configuration space associated to such a puzzle will tell you about solvability. In particular, if two configurations (say one solved $x$, one unsolved $y$) lie in the same path component of the configuration space, then the configuration $y$ can be solved.
      $endgroup$
      – Dan Rust
      Oct 29 '13 at 23:30










    • $begingroup$
      I've been solving these puzzles since I was a kid, and have never found one where the length of string is a limiting factor. If you solve puzzles like this for a while, you generally get the mindset that ball is out at infinity and don't worry about it.
      $endgroup$
      – Nick Alger
      Oct 30 '13 at 7:16












    • $begingroup$
      Imagine if the string in the puzzle above was shorter than the length needed for the loop to fit over one of the 'u-turns' at either end of the wire frame. The puzzle is now no longer solvable because one of the solving moves includes having to loop the string over one of those u-turns. The reason you've never seen a puzzle where it's a limiting factor is because no one would sell such a puzzle. Consider a puzzle with a free closed loop of string threaded through a tight metal ring. Now cap the loop off on either end with a large plastic ball - this is the unlink, but clearly unsolvable.
      $endgroup$
      – Dan Rust
      Oct 30 '13 at 11:03










    • $begingroup$
      I don't think that's a problem with the analogy.. If there is no homotopy from the unsolved to solved state, then it's unsolvable. If there is a homotopy, then either that homotopy is a solution, or it would be a solution but in real life the string is just too short.
      $endgroup$
      – Nick Alger
      Nov 2 '13 at 2:39



















    12












    $begingroup$

    I think it's very doable, I actually gave a short talk on topology to high school students once. Forget about the algebra and treat it very geometrically instead. Most importantly, draw lots of pictures! After all, it's really a visually intuitive subject, and even topologists spend a lot of time drawing pictures and convincing themselves about the geometry.



    Showcase some of the more immediately understandable results of the field - classification of surfaces, Brouwer fixed point theorem, hairy ball theorem, etc. and the cool examples - tori, Möbius band, Klein bottle, projective space, etc. Whenever possible, appeal to physical, visual, and geometric intuition.



    For applications, tell your audience that topology is a great reduction tool in many problems that at first seem complicated, examples being persistent homology, Morse theory, and topological degree theory. The IMA is having a special focus year on applications of algebraic topology (http://www.ima.umn.edu/2013-2014/) - check out the lectures! And make special mention of the computability of the theory - homology is a great tool because we can make the computer do it.



    Edit: I was speaking primarily about explaining topology to a general audience. I don't know much about homotopy groups, but if I were to say anything about fundamental groups I would make sure to mention the Poincare conjecture. It wouldn't hurt to mention how it had a USD 1 million bounty attached to it, and besides attempts at Poincare led us to some of the most fruitful mathematics of the past century, such as Morse theory.






    share|cite|improve this answer











    $endgroup$





















      7












      $begingroup$

      You might want to watch the talk of Tim Gowers "The Importance of Mathematics".



      In an nutshell, his answer to the politician was that some areas of mathematics have more practical applications than others but:




      1. It is very hard to predict which areas of mathematics will have applications in the future.

      2. The different areas of mathematics are very interconnected. Cutting the funding of a "non-profitable" area could harm the development of "profitable" ones.


      Of course he also talked about beauty and the quest for knowledge.






      share|cite|improve this answer









      $endgroup$





















        7












        $begingroup$

        I belive that you can find really interesting this stuff posted by Prof. Porter. It's not about the foundamental group but nonetheless present some real world application in algebraic topology (and there are some links to some documents too).



        That is an answer to a post of Tom Leinster addressing a similar question: he was looking for application of abstract math to motivate applied mathematicians into studying abstract maths and the opposite, find interesting application that motivates abstract maths for theoretical mathematicians.
        here the complete link.



        Anyway if you try to google a little bit you can find a lot of application of algebraic topology in applied field, in particular in data analysis and machine learning. If I'm not mistaken there are lots of work in persistent homology that have real world application. Unfortunately I don't have papers under my hand but try to look you'll most certain find lots of stuff.



        Now you could argue that these stuff doesn't directly regard fundamental group
        none the less to study such object it comes in handy to know a lot about fundamental groups. That's more lots of this applied tools were born in order to solve theoretical problems and later it was discovered they could be used into real world application. They are a sort of practice proof that theoretical research is as much important as applied one.



        Hope this very short and poor answer could help.
        Have good luck with you work :)



        Edit: After a while a was able to retrieve this old post on MathOverflow which I believe can be useful. There are some real world application of algebraic topology with some reference too.






        share|cite|improve this answer











        $endgroup$





















          5












          $begingroup$

          You might consider looking into topological mixing. One studies the optimal way you can mix, for instance, a viscous fluid with n-rods. This applies to such things as taffy pullers. Here are slides from a professor at my institution.






          share|cite|improve this answer









          $endgroup$





















            5












            $begingroup$

            Here is something that may be close to the politically minded hearts - the Human Heart! I have heard that methods of algebraic topology have been used in medical research of the human heart.




            1. When I was in grad school (late 80s), there was some documentary (may be just a news clip?) about them meds diagnosing imminent heart problems from the topology of the electric currents on the heart tissue. On the chalkboard behind the docs the calculations definitely included homotopy groups - don't remember whether it was the fundamental group. This was probably related to the knots formed by those currents, but obviously they didn't give the details. But something like: "the 2-torsion of the $n$th homotopy group of your heart currents changes => you will soon develop arrythmia (I'm making this up) would sound kinda convincing.

            2. When the technology of 3D-imaging of the human body was emerging, the people at the general hospital across the street from the sciences building asked a colleague the following question (he had helped them with compression algorithms and other handling of medical images before, so became the obvious target): the computer has stored a 3D-image of the heart of the patient as a 3D-array of small cubes, and can reliably tell whether each cube contains heart tissue or is empty (like part of an atrium or a ventricle). If that data is presented in a useful 3D-image, a trained human can quickly spot whether there are extra holes in the walls separating the chambers. But can this process be automated? We talked it over a lunch, and the best suggestion we came up with was to calculate the Euler characteristic of the 2D-surface formed by the boundary of the chambers (connectedness also). Surely the same solution had been proposed earlier elsewhere, but I think it was adopted (though there may be difficult to analyze border effects due to the granularity of the 3D-image). Anyway: letting the computer calculate the homology groups of the heart => the docs know whether heart surgery is needed doesn't sound too useless an application.


            Keeping this CW, because this is largely vague recollections, second (or third) hand information and such. Welcoming edits/links to references.






            share|cite|improve this answer











            $endgroup$









            • 2




              $begingroup$
              I remembered having read something about heart failure and topology a long time ago in the Scientific American and found this link.
              $endgroup$
              – WimC
              Oct 26 '13 at 19:17



















            5












            $begingroup$

            I worked in the classification of 3-manifolds. Briefly, here's what I came up with to explain this to people who were really interested. This doesn't so much justify the work politically, but it does give the general idea.




            • Imagine you're a two-dimensional creature. What could your world look like? It could be a plane. It could also be a sphere, which would make your life different because you always come back where you started. Or it could be the surface of a doughnut, which would be different again: there are two ways to loop around, and you know that they're different.
              The size of the sphere or doughnut doesn't matter, just the overall shape. The universe in the video game Asteroids is also a doughnut. This is "topology".


            • In fact there's an infinite list of universes: the plane, the sphere, the doughnut, doughnuts with any number of holes, some others which are twisted like a Moebius strip...


            • People solved this problem in the 19th century: we have a complete list of the two-dimensional universes, and if you give a two-dimensional universe, I can easily tell you which one it is. We have a "classification". The main tool for this classification is to use numbers to describe the shapes. That's "algebraic topology".


            • We seem to be living in a three-dimensional universe. What shape is it? What shapes could it be? Cosmologists, for example, would need to know this. But it turns out to be a much harder classification problem, and only partly solved. I worked on tools of algebraic topology to help with this problem.







            share|cite|improve this answer









            $endgroup$





















              4












              $begingroup$

              Have a look at the Knot Exhibition on this website and the applications there, including the history of knots, which seems to date back at least 300 millenia. One scientific application is to Knots and DNA, you can also do a web search on this. The design of the exhibition is to show how the work of understanding knots and their mathematics comes before you can make the applications. Tim Porter and I also did an article on Making a mathematical exhibition. See also Out of Line.



              The following picture shows a loop of string tied around part of a pentoil knot according to the formula



              $$xyxyxy^{-1} x^{-1} y^{-1} x^{-1} y^{-1} $$



              pent



              Then the loop of string comes off the knot! Explanation is in this seminar at Liverpool, for a demonstration this with a well travelled pentoil knot made of copper tubing and a length of thin rope.



              The idea for this trick comes from the determination of the fundamental group of the complement of the pentoil knot in $3$-space. See also pages 350-351 of Topology and Groupoids.






              share|cite|improve this answer











              $endgroup$





















                3












                $begingroup$

                I like the pictures from chapter 0 of Hatcher for that purpose.



                hatcher chapter 0



                And also Louis Kauffman made some nice pictures for the Fox calculus.



                fox calculus



                And Thurston's 3-manifolds book has good pictures as well.
                thurston



                "Normal maths works with squares, triangles, perfect circles; it counts beans. In topology we're freer to work with more interesting shapes." (cue picture from Dr Seuss)



                seuss



                This is basically what I did in http://tmblr.co/ZdCxIydt_Owi, drawing on the familiar philosophical distinction between reductivism vs holism to work in a cohomology element (as an argument for holism: some things can't be decomposed into atoms without losing some essential property that only shows up in how they're put together).



                A romantic notion I felt when I was first reading Hatcher is that the "counting rocks" and "straight-sight cannonball" problems that seem more typical of a more boring mathematics, get replaced by "feminine" "weaving-based" mathematics. (Women were the ones tasked with purling, picking, and wrapping in most of history.) So this is a nice tie-in for non-mathematicians, although I'm not sure AT actually is more "feminine" (whatever that means) it's an attention-grabber.





                You can also talk about topological spaces vs metric spaces by using highways



                USA road network



                versus airplane connections



                paris air connection



                taking a ring road versus trying to not back-track, versus taking small streets with different speed limits, could be thought of as metrics -- whereas if you're at the airport, then it's a question more of does the city you're in actually connect to where you're going or not, rather than 'physical' distance. You might end up flying past your destination (or at least extra distance) before taking a second flight/network hop, so the "network distance" is two (2 nbds), compare that to "ground distance". (Maybe for example Colorado Springs does not receive flights from L.A., so one has to go further east to Denver first ... if my geography is wrong then some pattern like what I'm suggesting does exist somewhere.)





                As far as applications, I like the suggestions of Ghrist & Carlsson's work. I would add Ghrist's robots-not-bumping-into-each-other in warehouses, anyone can easily envisage as a braid the robots not bumping into each other over time. (See Ghrist's EAT book ch 1 for the relevant picture.)



                ghrist braids robot motion planning factory floor



                But I would make a further point, which is that the goal is not to rehash the applications already developed, but rather why the novel approaches might lead to something new. In other words, "How has this already been applied?" and "How might this generate applications in the future?" are two separate questions.



                To that end I think it's fine, as long as you use clear non-technical language, to talk in generalities about how the "style" of topological thinking differs from maths they may be more familiar with. You could give some examples of "wiggly" or "non-rigid" situations and say that the ambition is that with enough development of AT, these situations could be handled.



                Justin Curry made a pitch like this in http://arxiv.org/abs/1303.3255, saying essentially "Linear algebra was amazing for applications; I think my thing could be similarly productive, from a different starting point."



                On the other hand, we probably all know examples of scientists who thought their research would be used for totally different purposes than what it ended up in. My favourite example is the inventor of NMR (worth a Nobel prize) thought NMR would be used for "calibrating industrial magnets". Instead it was used to look inside people's brains. (But that was someone else's work, to apply it to that.)



                On the other other hand, probably no politicians slapped Purcell & Bloch on the wrist when they said they were working on technology to calibrate industrial magnets. That can be envisaged by a politician and sounds science-y and maybe-important (just not as important as what it ended up being). Some mathematicians have also been known to name their ideas for marketing purposes, for example "dynamic programming" and "information entropy". (There are funny stories going along with the naming of each of these.)





                If I had a piece of paper, my 10-second elevator pitch for algebraic topology would be:




                1. Most data-analysis methods assume data is "Square" (meaning Euclidean--independent dimensions, flat, etc)

                2. But how do we know that? What if the data is shaped like a peace sign? pax-benz

                3. We don't want to be caught off-guard by assumptions we weren't necessarily trying to make, so do AT to be aware of other possibilities the world might hit you with.


                thor vigfursson






                share|cite|improve this answer











                $endgroup$





















                  2












                  $begingroup$

                  The following examples come to mind. Show that the real projective plane, i.e., a Möbius strip glued with a disc along their boundaries, cannot be embedded in ${mathbb R}^3$. In its turn, you can additionally show that the Möbius strip (without boundary) is the space of all non-ordered pairs of points on a circle. There are many ways to prove these two statements, and some are quite elementary. I believe some proofs can be performed almost without words, just using pictures (well, elaborating pictures requires some serious work). You might take one and try to adapt it for "politicians". Good luck!






                  share|cite|improve this answer









                  $endgroup$





















                    1












                    $begingroup$

                    Roger Penrose in some of his lectures challenged students to find an everyday accessible example best described by second or higher cohomology.



                    I would construct an functional or relational description of a dynamic system and make some predictions about it using algebraic topology. Then you can use ordinary language. Try modelling two administrative systems using only relations of parts and show their topological equivalence, if it has to convince politicians...



                    I have some examples, but they are much too complicated to fit on one or two pages if stated verbally. And they might make you fail, depending on what politics your professor has, so be careful to choose examples which people don't have strong feelings about, if doing this.






                    share|cite|improve this answer









                    $endgroup$














                      Your Answer








                      StackExchange.ready(function() {
                      var channelOptions = {
                      tags: "".split(" "),
                      id: "69"
                      };
                      initTagRenderer("".split(" "), "".split(" "), channelOptions);

                      StackExchange.using("externalEditor", function() {
                      // Have to fire editor after snippets, if snippets enabled
                      if (StackExchange.settings.snippets.snippetsEnabled) {
                      StackExchange.using("snippets", function() {
                      createEditor();
                      });
                      }
                      else {
                      createEditor();
                      }
                      });

                      function createEditor() {
                      StackExchange.prepareEditor({
                      heartbeatType: 'answer',
                      autoActivateHeartbeat: false,
                      convertImagesToLinks: true,
                      noModals: true,
                      showLowRepImageUploadWarning: true,
                      reputationToPostImages: 10,
                      bindNavPrevention: true,
                      postfix: "",
                      imageUploader: {
                      brandingHtml: "Powered by u003ca class="icon-imgur-white" href="https://imgur.com/"u003eu003c/au003e",
                      contentPolicyHtml: "User contributions licensed under u003ca href="https://creativecommons.org/licenses/by-sa/3.0/"u003ecc by-sa 3.0 with attribution requiredu003c/au003e u003ca href="https://stackoverflow.com/legal/content-policy"u003e(content policy)u003c/au003e",
                      allowUrls: true
                      },
                      noCode: true, onDemand: true,
                      discardSelector: ".discard-answer"
                      ,immediatelyShowMarkdownHelp:true
                      });


                      }
                      });














                      draft saved

                      draft discarded


















                      StackExchange.ready(
                      function () {
                      StackExchange.openid.initPostLogin('.new-post-login', 'https%3a%2f%2fmath.stackexchange.com%2fquestions%2f539675%2fhow-to-present-algebraic-topology-to-non-mathematicians%23new-answer', 'question_page');
                      }
                      );

                      Post as a guest















                      Required, but never shown

























                      13 Answers
                      13






                      active

                      oldest

                      votes








                      13 Answers
                      13






                      active

                      oldest

                      votes









                      active

                      oldest

                      votes






                      active

                      oldest

                      votes









                      28












                      $begingroup$

                      Here are two examples of algebraic topology (actually homology theory) being applied to "real world" problems.



                      1) Ghrist's paper "Homological sensor networks." (pdf)



                      2) Topological data analysis, as pioneered by Gunnar Carlsson.






                      share|cite|improve this answer











                      $endgroup$


















                        28












                        $begingroup$

                        Here are two examples of algebraic topology (actually homology theory) being applied to "real world" problems.



                        1) Ghrist's paper "Homological sensor networks." (pdf)



                        2) Topological data analysis, as pioneered by Gunnar Carlsson.






                        share|cite|improve this answer











                        $endgroup$
















                          28












                          28








                          28





                          $begingroup$

                          Here are two examples of algebraic topology (actually homology theory) being applied to "real world" problems.



                          1) Ghrist's paper "Homological sensor networks." (pdf)



                          2) Topological data analysis, as pioneered by Gunnar Carlsson.






                          share|cite|improve this answer











                          $endgroup$



                          Here are two examples of algebraic topology (actually homology theory) being applied to "real world" problems.



                          1) Ghrist's paper "Homological sensor networks." (pdf)



                          2) Topological data analysis, as pioneered by Gunnar Carlsson.







                          share|cite|improve this answer














                          share|cite|improve this answer



                          share|cite|improve this answer








                          edited Aug 27 '14 at 22:50









                          Sasha

                          61.1k5110183




                          61.1k5110183










                          answered Oct 25 '13 at 23:42









                          Cheerful ParsnipCheerful Parsnip

                          21.2k23599




                          21.2k23599























                              20












                              $begingroup$

                              Explain that in a society that values learning, the pursuit of abstract knowledge is an end unto itself. This is how pure mathematicians have answered that question for millennia and is, as far as I know, the only truly irrefutable answer.






                              share|cite|improve this answer









                              $endgroup$









                              • 14




                                $begingroup$
                                Not very helpful. Sure abstract knowledge is worth pursuing, but you're generally a bit more selective about what abstract knowledge you pursue. If I made up a meaningless formal system of axioms right now, and then acted condescending towards people who didn't find them interesting, I'd be rightly considered mad. What aspects of algebraic topology make its abstract study interesting?
                                $endgroup$
                                – Jack M
                                Oct 26 '13 at 0:18






                              • 3




                                $begingroup$
                                The OP specifically mentioned reasons for explaining algebraic topology to laymen: to get financial support for his work, and to motivate students. I don't think this answer does a good job at achieving either goal. This may be an irrefutable answer, but it's not a useful one.
                                $endgroup$
                                – Gyu Eun Lee
                                Oct 26 '13 at 4:31






                              • 9




                                $begingroup$
                                I think there are some really cool arguments to be had here. One of my favorite recent articles is called "Understanding is a Poor Substitute for Convexity" and it argues that science is productive simply because it is able to create convex payoff functions, and that supporting research at arbitrarily and at random is already quite a good strategy, and maybe even near-optimal. Together with the philosophical sentiment that we should be a society that values learning—well, it's compelling. I love it. But it is most certainly not a way to get any particular project funded.
                                $endgroup$
                                – Slade
                                Oct 26 '13 at 8:48






                              • 2




                                $begingroup$
                                This answer only tells me that you're doing something for which you don't see any practical application. It doesn't tell me what you're doing, and it doesn't tell me that your pursuit has any kind of long-term goal or structure. Are you doing anything other than intellectual masturbation all day?
                                $endgroup$
                                – Jonas
                                Oct 26 '13 at 13:14






                              • 1




                                $begingroup$
                                @Jonas That argument serves equally well to defund scholars in the philosophy department, the classics department, the English department, the art history department, and even the astronomy department. Its wide applicability betrays its emptiness. Yes, scholarship can seem like intellectual masturbation to outsiders, but that observation is neither recent nor intelligent.
                                $endgroup$
                                – TBrendle
                                Oct 28 '13 at 3:39


















                              20












                              $begingroup$

                              Explain that in a society that values learning, the pursuit of abstract knowledge is an end unto itself. This is how pure mathematicians have answered that question for millennia and is, as far as I know, the only truly irrefutable answer.






                              share|cite|improve this answer









                              $endgroup$









                              • 14




                                $begingroup$
                                Not very helpful. Sure abstract knowledge is worth pursuing, but you're generally a bit more selective about what abstract knowledge you pursue. If I made up a meaningless formal system of axioms right now, and then acted condescending towards people who didn't find them interesting, I'd be rightly considered mad. What aspects of algebraic topology make its abstract study interesting?
                                $endgroup$
                                – Jack M
                                Oct 26 '13 at 0:18






                              • 3




                                $begingroup$
                                The OP specifically mentioned reasons for explaining algebraic topology to laymen: to get financial support for his work, and to motivate students. I don't think this answer does a good job at achieving either goal. This may be an irrefutable answer, but it's not a useful one.
                                $endgroup$
                                – Gyu Eun Lee
                                Oct 26 '13 at 4:31






                              • 9




                                $begingroup$
                                I think there are some really cool arguments to be had here. One of my favorite recent articles is called "Understanding is a Poor Substitute for Convexity" and it argues that science is productive simply because it is able to create convex payoff functions, and that supporting research at arbitrarily and at random is already quite a good strategy, and maybe even near-optimal. Together with the philosophical sentiment that we should be a society that values learning—well, it's compelling. I love it. But it is most certainly not a way to get any particular project funded.
                                $endgroup$
                                – Slade
                                Oct 26 '13 at 8:48






                              • 2




                                $begingroup$
                                This answer only tells me that you're doing something for which you don't see any practical application. It doesn't tell me what you're doing, and it doesn't tell me that your pursuit has any kind of long-term goal or structure. Are you doing anything other than intellectual masturbation all day?
                                $endgroup$
                                – Jonas
                                Oct 26 '13 at 13:14






                              • 1




                                $begingroup$
                                @Jonas That argument serves equally well to defund scholars in the philosophy department, the classics department, the English department, the art history department, and even the astronomy department. Its wide applicability betrays its emptiness. Yes, scholarship can seem like intellectual masturbation to outsiders, but that observation is neither recent nor intelligent.
                                $endgroup$
                                – TBrendle
                                Oct 28 '13 at 3:39
















                              20












                              20








                              20





                              $begingroup$

                              Explain that in a society that values learning, the pursuit of abstract knowledge is an end unto itself. This is how pure mathematicians have answered that question for millennia and is, as far as I know, the only truly irrefutable answer.






                              share|cite|improve this answer









                              $endgroup$



                              Explain that in a society that values learning, the pursuit of abstract knowledge is an end unto itself. This is how pure mathematicians have answered that question for millennia and is, as far as I know, the only truly irrefutable answer.







                              share|cite|improve this answer












                              share|cite|improve this answer



                              share|cite|improve this answer










                              answered Oct 25 '13 at 20:48









                              TBrendleTBrendle

                              2,8931017




                              2,8931017








                              • 14




                                $begingroup$
                                Not very helpful. Sure abstract knowledge is worth pursuing, but you're generally a bit more selective about what abstract knowledge you pursue. If I made up a meaningless formal system of axioms right now, and then acted condescending towards people who didn't find them interesting, I'd be rightly considered mad. What aspects of algebraic topology make its abstract study interesting?
                                $endgroup$
                                – Jack M
                                Oct 26 '13 at 0:18






                              • 3




                                $begingroup$
                                The OP specifically mentioned reasons for explaining algebraic topology to laymen: to get financial support for his work, and to motivate students. I don't think this answer does a good job at achieving either goal. This may be an irrefutable answer, but it's not a useful one.
                                $endgroup$
                                – Gyu Eun Lee
                                Oct 26 '13 at 4:31






                              • 9




                                $begingroup$
                                I think there are some really cool arguments to be had here. One of my favorite recent articles is called "Understanding is a Poor Substitute for Convexity" and it argues that science is productive simply because it is able to create convex payoff functions, and that supporting research at arbitrarily and at random is already quite a good strategy, and maybe even near-optimal. Together with the philosophical sentiment that we should be a society that values learning—well, it's compelling. I love it. But it is most certainly not a way to get any particular project funded.
                                $endgroup$
                                – Slade
                                Oct 26 '13 at 8:48






                              • 2




                                $begingroup$
                                This answer only tells me that you're doing something for which you don't see any practical application. It doesn't tell me what you're doing, and it doesn't tell me that your pursuit has any kind of long-term goal or structure. Are you doing anything other than intellectual masturbation all day?
                                $endgroup$
                                – Jonas
                                Oct 26 '13 at 13:14






                              • 1




                                $begingroup$
                                @Jonas That argument serves equally well to defund scholars in the philosophy department, the classics department, the English department, the art history department, and even the astronomy department. Its wide applicability betrays its emptiness. Yes, scholarship can seem like intellectual masturbation to outsiders, but that observation is neither recent nor intelligent.
                                $endgroup$
                                – TBrendle
                                Oct 28 '13 at 3:39
















                              • 14




                                $begingroup$
                                Not very helpful. Sure abstract knowledge is worth pursuing, but you're generally a bit more selective about what abstract knowledge you pursue. If I made up a meaningless formal system of axioms right now, and then acted condescending towards people who didn't find them interesting, I'd be rightly considered mad. What aspects of algebraic topology make its abstract study interesting?
                                $endgroup$
                                – Jack M
                                Oct 26 '13 at 0:18






                              • 3




                                $begingroup$
                                The OP specifically mentioned reasons for explaining algebraic topology to laymen: to get financial support for his work, and to motivate students. I don't think this answer does a good job at achieving either goal. This may be an irrefutable answer, but it's not a useful one.
                                $endgroup$
                                – Gyu Eun Lee
                                Oct 26 '13 at 4:31






                              • 9




                                $begingroup$
                                I think there are some really cool arguments to be had here. One of my favorite recent articles is called "Understanding is a Poor Substitute for Convexity" and it argues that science is productive simply because it is able to create convex payoff functions, and that supporting research at arbitrarily and at random is already quite a good strategy, and maybe even near-optimal. Together with the philosophical sentiment that we should be a society that values learning—well, it's compelling. I love it. But it is most certainly not a way to get any particular project funded.
                                $endgroup$
                                – Slade
                                Oct 26 '13 at 8:48






                              • 2




                                $begingroup$
                                This answer only tells me that you're doing something for which you don't see any practical application. It doesn't tell me what you're doing, and it doesn't tell me that your pursuit has any kind of long-term goal or structure. Are you doing anything other than intellectual masturbation all day?
                                $endgroup$
                                – Jonas
                                Oct 26 '13 at 13:14






                              • 1




                                $begingroup$
                                @Jonas That argument serves equally well to defund scholars in the philosophy department, the classics department, the English department, the art history department, and even the astronomy department. Its wide applicability betrays its emptiness. Yes, scholarship can seem like intellectual masturbation to outsiders, but that observation is neither recent nor intelligent.
                                $endgroup$
                                – TBrendle
                                Oct 28 '13 at 3:39










                              14




                              14




                              $begingroup$
                              Not very helpful. Sure abstract knowledge is worth pursuing, but you're generally a bit more selective about what abstract knowledge you pursue. If I made up a meaningless formal system of axioms right now, and then acted condescending towards people who didn't find them interesting, I'd be rightly considered mad. What aspects of algebraic topology make its abstract study interesting?
                              $endgroup$
                              – Jack M
                              Oct 26 '13 at 0:18




                              $begingroup$
                              Not very helpful. Sure abstract knowledge is worth pursuing, but you're generally a bit more selective about what abstract knowledge you pursue. If I made up a meaningless formal system of axioms right now, and then acted condescending towards people who didn't find them interesting, I'd be rightly considered mad. What aspects of algebraic topology make its abstract study interesting?
                              $endgroup$
                              – Jack M
                              Oct 26 '13 at 0:18




                              3




                              3




                              $begingroup$
                              The OP specifically mentioned reasons for explaining algebraic topology to laymen: to get financial support for his work, and to motivate students. I don't think this answer does a good job at achieving either goal. This may be an irrefutable answer, but it's not a useful one.
                              $endgroup$
                              – Gyu Eun Lee
                              Oct 26 '13 at 4:31




                              $begingroup$
                              The OP specifically mentioned reasons for explaining algebraic topology to laymen: to get financial support for his work, and to motivate students. I don't think this answer does a good job at achieving either goal. This may be an irrefutable answer, but it's not a useful one.
                              $endgroup$
                              – Gyu Eun Lee
                              Oct 26 '13 at 4:31




                              9




                              9




                              $begingroup$
                              I think there are some really cool arguments to be had here. One of my favorite recent articles is called "Understanding is a Poor Substitute for Convexity" and it argues that science is productive simply because it is able to create convex payoff functions, and that supporting research at arbitrarily and at random is already quite a good strategy, and maybe even near-optimal. Together with the philosophical sentiment that we should be a society that values learning—well, it's compelling. I love it. But it is most certainly not a way to get any particular project funded.
                              $endgroup$
                              – Slade
                              Oct 26 '13 at 8:48




                              $begingroup$
                              I think there are some really cool arguments to be had here. One of my favorite recent articles is called "Understanding is a Poor Substitute for Convexity" and it argues that science is productive simply because it is able to create convex payoff functions, and that supporting research at arbitrarily and at random is already quite a good strategy, and maybe even near-optimal. Together with the philosophical sentiment that we should be a society that values learning—well, it's compelling. I love it. But it is most certainly not a way to get any particular project funded.
                              $endgroup$
                              – Slade
                              Oct 26 '13 at 8:48




                              2




                              2




                              $begingroup$
                              This answer only tells me that you're doing something for which you don't see any practical application. It doesn't tell me what you're doing, and it doesn't tell me that your pursuit has any kind of long-term goal or structure. Are you doing anything other than intellectual masturbation all day?
                              $endgroup$
                              – Jonas
                              Oct 26 '13 at 13:14




                              $begingroup$
                              This answer only tells me that you're doing something for which you don't see any practical application. It doesn't tell me what you're doing, and it doesn't tell me that your pursuit has any kind of long-term goal or structure. Are you doing anything other than intellectual masturbation all day?
                              $endgroup$
                              – Jonas
                              Oct 26 '13 at 13:14




                              1




                              1




                              $begingroup$
                              @Jonas That argument serves equally well to defund scholars in the philosophy department, the classics department, the English department, the art history department, and even the astronomy department. Its wide applicability betrays its emptiness. Yes, scholarship can seem like intellectual masturbation to outsiders, but that observation is neither recent nor intelligent.
                              $endgroup$
                              – TBrendle
                              Oct 28 '13 at 3:39






                              $begingroup$
                              @Jonas That argument serves equally well to defund scholars in the philosophy department, the classics department, the English department, the art history department, and even the astronomy department. Its wide applicability betrays its emptiness. Yes, scholarship can seem like intellectual masturbation to outsiders, but that observation is neither recent nor intelligent.
                              $endgroup$
                              – TBrendle
                              Oct 28 '13 at 3:39













                              15












                              $begingroup$

                              It may not be "useful", but you can explain the subject to a nontechnical audience by relating it to "wire and string" puzzles. The solvability of wire-and-string puzzles can basically be reduced to questions about the fundamental group of the complementary ambient space surrounding the metal wire.



                              For example, look at the following "astroknot puzzle", where the goal is to remove the long flexible string from the solid metal,
                              enter image description here



                              This also, incidentally, leads to one of the best ways to solve such puzzles as a human, which is to imagine that the inflexible metal can bend into a form where the puzzle is easy, and then imagine how the nearby air would deform under the reverse transformation.






                              share|cite|improve this answer











                              $endgroup$









                              • 2




                                $begingroup$
                                I don't think you're being totally genuine by saying the solvability of such a puzzle is only determined by the topology of the ambient complement. In such puzzles, the lengths of both the strings, and the shapes/lengths of the rigid parts can affect whether the puzzle is solvable. It is true though that the topology of the configuration space associated to such a puzzle will tell you about solvability. In particular, if two configurations (say one solved $x$, one unsolved $y$) lie in the same path component of the configuration space, then the configuration $y$ can be solved.
                                $endgroup$
                                – Dan Rust
                                Oct 29 '13 at 23:30










                              • $begingroup$
                                I've been solving these puzzles since I was a kid, and have never found one where the length of string is a limiting factor. If you solve puzzles like this for a while, you generally get the mindset that ball is out at infinity and don't worry about it.
                                $endgroup$
                                – Nick Alger
                                Oct 30 '13 at 7:16












                              • $begingroup$
                                Imagine if the string in the puzzle above was shorter than the length needed for the loop to fit over one of the 'u-turns' at either end of the wire frame. The puzzle is now no longer solvable because one of the solving moves includes having to loop the string over one of those u-turns. The reason you've never seen a puzzle where it's a limiting factor is because no one would sell such a puzzle. Consider a puzzle with a free closed loop of string threaded through a tight metal ring. Now cap the loop off on either end with a large plastic ball - this is the unlink, but clearly unsolvable.
                                $endgroup$
                                – Dan Rust
                                Oct 30 '13 at 11:03










                              • $begingroup$
                                I don't think that's a problem with the analogy.. If there is no homotopy from the unsolved to solved state, then it's unsolvable. If there is a homotopy, then either that homotopy is a solution, or it would be a solution but in real life the string is just too short.
                                $endgroup$
                                – Nick Alger
                                Nov 2 '13 at 2:39
















                              15












                              $begingroup$

                              It may not be "useful", but you can explain the subject to a nontechnical audience by relating it to "wire and string" puzzles. The solvability of wire-and-string puzzles can basically be reduced to questions about the fundamental group of the complementary ambient space surrounding the metal wire.



                              For example, look at the following "astroknot puzzle", where the goal is to remove the long flexible string from the solid metal,
                              enter image description here



                              This also, incidentally, leads to one of the best ways to solve such puzzles as a human, which is to imagine that the inflexible metal can bend into a form where the puzzle is easy, and then imagine how the nearby air would deform under the reverse transformation.






                              share|cite|improve this answer











                              $endgroup$









                              • 2




                                $begingroup$
                                I don't think you're being totally genuine by saying the solvability of such a puzzle is only determined by the topology of the ambient complement. In such puzzles, the lengths of both the strings, and the shapes/lengths of the rigid parts can affect whether the puzzle is solvable. It is true though that the topology of the configuration space associated to such a puzzle will tell you about solvability. In particular, if two configurations (say one solved $x$, one unsolved $y$) lie in the same path component of the configuration space, then the configuration $y$ can be solved.
                                $endgroup$
                                – Dan Rust
                                Oct 29 '13 at 23:30










                              • $begingroup$
                                I've been solving these puzzles since I was a kid, and have never found one where the length of string is a limiting factor. If you solve puzzles like this for a while, you generally get the mindset that ball is out at infinity and don't worry about it.
                                $endgroup$
                                – Nick Alger
                                Oct 30 '13 at 7:16












                              • $begingroup$
                                Imagine if the string in the puzzle above was shorter than the length needed for the loop to fit over one of the 'u-turns' at either end of the wire frame. The puzzle is now no longer solvable because one of the solving moves includes having to loop the string over one of those u-turns. The reason you've never seen a puzzle where it's a limiting factor is because no one would sell such a puzzle. Consider a puzzle with a free closed loop of string threaded through a tight metal ring. Now cap the loop off on either end with a large plastic ball - this is the unlink, but clearly unsolvable.
                                $endgroup$
                                – Dan Rust
                                Oct 30 '13 at 11:03










                              • $begingroup$
                                I don't think that's a problem with the analogy.. If there is no homotopy from the unsolved to solved state, then it's unsolvable. If there is a homotopy, then either that homotopy is a solution, or it would be a solution but in real life the string is just too short.
                                $endgroup$
                                – Nick Alger
                                Nov 2 '13 at 2:39














                              15












                              15








                              15





                              $begingroup$

                              It may not be "useful", but you can explain the subject to a nontechnical audience by relating it to "wire and string" puzzles. The solvability of wire-and-string puzzles can basically be reduced to questions about the fundamental group of the complementary ambient space surrounding the metal wire.



                              For example, look at the following "astroknot puzzle", where the goal is to remove the long flexible string from the solid metal,
                              enter image description here



                              This also, incidentally, leads to one of the best ways to solve such puzzles as a human, which is to imagine that the inflexible metal can bend into a form where the puzzle is easy, and then imagine how the nearby air would deform under the reverse transformation.






                              share|cite|improve this answer











                              $endgroup$



                              It may not be "useful", but you can explain the subject to a nontechnical audience by relating it to "wire and string" puzzles. The solvability of wire-and-string puzzles can basically be reduced to questions about the fundamental group of the complementary ambient space surrounding the metal wire.



                              For example, look at the following "astroknot puzzle", where the goal is to remove the long flexible string from the solid metal,
                              enter image description here



                              This also, incidentally, leads to one of the best ways to solve such puzzles as a human, which is to imagine that the inflexible metal can bend into a form where the puzzle is easy, and then imagine how the nearby air would deform under the reverse transformation.







                              share|cite|improve this answer














                              share|cite|improve this answer



                              share|cite|improve this answer








                              edited Oct 26 '13 at 6:53

























                              answered Oct 25 '13 at 21:53









                              Nick AlgerNick Alger

                              10.3k63369




                              10.3k63369








                              • 2




                                $begingroup$
                                I don't think you're being totally genuine by saying the solvability of such a puzzle is only determined by the topology of the ambient complement. In such puzzles, the lengths of both the strings, and the shapes/lengths of the rigid parts can affect whether the puzzle is solvable. It is true though that the topology of the configuration space associated to such a puzzle will tell you about solvability. In particular, if two configurations (say one solved $x$, one unsolved $y$) lie in the same path component of the configuration space, then the configuration $y$ can be solved.
                                $endgroup$
                                – Dan Rust
                                Oct 29 '13 at 23:30










                              • $begingroup$
                                I've been solving these puzzles since I was a kid, and have never found one where the length of string is a limiting factor. If you solve puzzles like this for a while, you generally get the mindset that ball is out at infinity and don't worry about it.
                                $endgroup$
                                – Nick Alger
                                Oct 30 '13 at 7:16












                              • $begingroup$
                                Imagine if the string in the puzzle above was shorter than the length needed for the loop to fit over one of the 'u-turns' at either end of the wire frame. The puzzle is now no longer solvable because one of the solving moves includes having to loop the string over one of those u-turns. The reason you've never seen a puzzle where it's a limiting factor is because no one would sell such a puzzle. Consider a puzzle with a free closed loop of string threaded through a tight metal ring. Now cap the loop off on either end with a large plastic ball - this is the unlink, but clearly unsolvable.
                                $endgroup$
                                – Dan Rust
                                Oct 30 '13 at 11:03










                              • $begingroup$
                                I don't think that's a problem with the analogy.. If there is no homotopy from the unsolved to solved state, then it's unsolvable. If there is a homotopy, then either that homotopy is a solution, or it would be a solution but in real life the string is just too short.
                                $endgroup$
                                – Nick Alger
                                Nov 2 '13 at 2:39














                              • 2




                                $begingroup$
                                I don't think you're being totally genuine by saying the solvability of such a puzzle is only determined by the topology of the ambient complement. In such puzzles, the lengths of both the strings, and the shapes/lengths of the rigid parts can affect whether the puzzle is solvable. It is true though that the topology of the configuration space associated to such a puzzle will tell you about solvability. In particular, if two configurations (say one solved $x$, one unsolved $y$) lie in the same path component of the configuration space, then the configuration $y$ can be solved.
                                $endgroup$
                                – Dan Rust
                                Oct 29 '13 at 23:30










                              • $begingroup$
                                I've been solving these puzzles since I was a kid, and have never found one where the length of string is a limiting factor. If you solve puzzles like this for a while, you generally get the mindset that ball is out at infinity and don't worry about it.
                                $endgroup$
                                – Nick Alger
                                Oct 30 '13 at 7:16












                              • $begingroup$
                                Imagine if the string in the puzzle above was shorter than the length needed for the loop to fit over one of the 'u-turns' at either end of the wire frame. The puzzle is now no longer solvable because one of the solving moves includes having to loop the string over one of those u-turns. The reason you've never seen a puzzle where it's a limiting factor is because no one would sell such a puzzle. Consider a puzzle with a free closed loop of string threaded through a tight metal ring. Now cap the loop off on either end with a large plastic ball - this is the unlink, but clearly unsolvable.
                                $endgroup$
                                – Dan Rust
                                Oct 30 '13 at 11:03










                              • $begingroup$
                                I don't think that's a problem with the analogy.. If there is no homotopy from the unsolved to solved state, then it's unsolvable. If there is a homotopy, then either that homotopy is a solution, or it would be a solution but in real life the string is just too short.
                                $endgroup$
                                – Nick Alger
                                Nov 2 '13 at 2:39








                              2




                              2




                              $begingroup$
                              I don't think you're being totally genuine by saying the solvability of such a puzzle is only determined by the topology of the ambient complement. In such puzzles, the lengths of both the strings, and the shapes/lengths of the rigid parts can affect whether the puzzle is solvable. It is true though that the topology of the configuration space associated to such a puzzle will tell you about solvability. In particular, if two configurations (say one solved $x$, one unsolved $y$) lie in the same path component of the configuration space, then the configuration $y$ can be solved.
                              $endgroup$
                              – Dan Rust
                              Oct 29 '13 at 23:30




                              $begingroup$
                              I don't think you're being totally genuine by saying the solvability of such a puzzle is only determined by the topology of the ambient complement. In such puzzles, the lengths of both the strings, and the shapes/lengths of the rigid parts can affect whether the puzzle is solvable. It is true though that the topology of the configuration space associated to such a puzzle will tell you about solvability. In particular, if two configurations (say one solved $x$, one unsolved $y$) lie in the same path component of the configuration space, then the configuration $y$ can be solved.
                              $endgroup$
                              – Dan Rust
                              Oct 29 '13 at 23:30












                              $begingroup$
                              I've been solving these puzzles since I was a kid, and have never found one where the length of string is a limiting factor. If you solve puzzles like this for a while, you generally get the mindset that ball is out at infinity and don't worry about it.
                              $endgroup$
                              – Nick Alger
                              Oct 30 '13 at 7:16






                              $begingroup$
                              I've been solving these puzzles since I was a kid, and have never found one where the length of string is a limiting factor. If you solve puzzles like this for a while, you generally get the mindset that ball is out at infinity and don't worry about it.
                              $endgroup$
                              – Nick Alger
                              Oct 30 '13 at 7:16














                              $begingroup$
                              Imagine if the string in the puzzle above was shorter than the length needed for the loop to fit over one of the 'u-turns' at either end of the wire frame. The puzzle is now no longer solvable because one of the solving moves includes having to loop the string over one of those u-turns. The reason you've never seen a puzzle where it's a limiting factor is because no one would sell such a puzzle. Consider a puzzle with a free closed loop of string threaded through a tight metal ring. Now cap the loop off on either end with a large plastic ball - this is the unlink, but clearly unsolvable.
                              $endgroup$
                              – Dan Rust
                              Oct 30 '13 at 11:03




                              $begingroup$
                              Imagine if the string in the puzzle above was shorter than the length needed for the loop to fit over one of the 'u-turns' at either end of the wire frame. The puzzle is now no longer solvable because one of the solving moves includes having to loop the string over one of those u-turns. The reason you've never seen a puzzle where it's a limiting factor is because no one would sell such a puzzle. Consider a puzzle with a free closed loop of string threaded through a tight metal ring. Now cap the loop off on either end with a large plastic ball - this is the unlink, but clearly unsolvable.
                              $endgroup$
                              – Dan Rust
                              Oct 30 '13 at 11:03












                              $begingroup$
                              I don't think that's a problem with the analogy.. If there is no homotopy from the unsolved to solved state, then it's unsolvable. If there is a homotopy, then either that homotopy is a solution, or it would be a solution but in real life the string is just too short.
                              $endgroup$
                              – Nick Alger
                              Nov 2 '13 at 2:39




                              $begingroup$
                              I don't think that's a problem with the analogy.. If there is no homotopy from the unsolved to solved state, then it's unsolvable. If there is a homotopy, then either that homotopy is a solution, or it would be a solution but in real life the string is just too short.
                              $endgroup$
                              – Nick Alger
                              Nov 2 '13 at 2:39











                              12












                              $begingroup$

                              I think it's very doable, I actually gave a short talk on topology to high school students once. Forget about the algebra and treat it very geometrically instead. Most importantly, draw lots of pictures! After all, it's really a visually intuitive subject, and even topologists spend a lot of time drawing pictures and convincing themselves about the geometry.



                              Showcase some of the more immediately understandable results of the field - classification of surfaces, Brouwer fixed point theorem, hairy ball theorem, etc. and the cool examples - tori, Möbius band, Klein bottle, projective space, etc. Whenever possible, appeal to physical, visual, and geometric intuition.



                              For applications, tell your audience that topology is a great reduction tool in many problems that at first seem complicated, examples being persistent homology, Morse theory, and topological degree theory. The IMA is having a special focus year on applications of algebraic topology (http://www.ima.umn.edu/2013-2014/) - check out the lectures! And make special mention of the computability of the theory - homology is a great tool because we can make the computer do it.



                              Edit: I was speaking primarily about explaining topology to a general audience. I don't know much about homotopy groups, but if I were to say anything about fundamental groups I would make sure to mention the Poincare conjecture. It wouldn't hurt to mention how it had a USD 1 million bounty attached to it, and besides attempts at Poincare led us to some of the most fruitful mathematics of the past century, such as Morse theory.






                              share|cite|improve this answer











                              $endgroup$


















                                12












                                $begingroup$

                                I think it's very doable, I actually gave a short talk on topology to high school students once. Forget about the algebra and treat it very geometrically instead. Most importantly, draw lots of pictures! After all, it's really a visually intuitive subject, and even topologists spend a lot of time drawing pictures and convincing themselves about the geometry.



                                Showcase some of the more immediately understandable results of the field - classification of surfaces, Brouwer fixed point theorem, hairy ball theorem, etc. and the cool examples - tori, Möbius band, Klein bottle, projective space, etc. Whenever possible, appeal to physical, visual, and geometric intuition.



                                For applications, tell your audience that topology is a great reduction tool in many problems that at first seem complicated, examples being persistent homology, Morse theory, and topological degree theory. The IMA is having a special focus year on applications of algebraic topology (http://www.ima.umn.edu/2013-2014/) - check out the lectures! And make special mention of the computability of the theory - homology is a great tool because we can make the computer do it.



                                Edit: I was speaking primarily about explaining topology to a general audience. I don't know much about homotopy groups, but if I were to say anything about fundamental groups I would make sure to mention the Poincare conjecture. It wouldn't hurt to mention how it had a USD 1 million bounty attached to it, and besides attempts at Poincare led us to some of the most fruitful mathematics of the past century, such as Morse theory.






                                share|cite|improve this answer











                                $endgroup$
















                                  12












                                  12








                                  12





                                  $begingroup$

                                  I think it's very doable, I actually gave a short talk on topology to high school students once. Forget about the algebra and treat it very geometrically instead. Most importantly, draw lots of pictures! After all, it's really a visually intuitive subject, and even topologists spend a lot of time drawing pictures and convincing themselves about the geometry.



                                  Showcase some of the more immediately understandable results of the field - classification of surfaces, Brouwer fixed point theorem, hairy ball theorem, etc. and the cool examples - tori, Möbius band, Klein bottle, projective space, etc. Whenever possible, appeal to physical, visual, and geometric intuition.



                                  For applications, tell your audience that topology is a great reduction tool in many problems that at first seem complicated, examples being persistent homology, Morse theory, and topological degree theory. The IMA is having a special focus year on applications of algebraic topology (http://www.ima.umn.edu/2013-2014/) - check out the lectures! And make special mention of the computability of the theory - homology is a great tool because we can make the computer do it.



                                  Edit: I was speaking primarily about explaining topology to a general audience. I don't know much about homotopy groups, but if I were to say anything about fundamental groups I would make sure to mention the Poincare conjecture. It wouldn't hurt to mention how it had a USD 1 million bounty attached to it, and besides attempts at Poincare led us to some of the most fruitful mathematics of the past century, such as Morse theory.






                                  share|cite|improve this answer











                                  $endgroup$



                                  I think it's very doable, I actually gave a short talk on topology to high school students once. Forget about the algebra and treat it very geometrically instead. Most importantly, draw lots of pictures! After all, it's really a visually intuitive subject, and even topologists spend a lot of time drawing pictures and convincing themselves about the geometry.



                                  Showcase some of the more immediately understandable results of the field - classification of surfaces, Brouwer fixed point theorem, hairy ball theorem, etc. and the cool examples - tori, Möbius band, Klein bottle, projective space, etc. Whenever possible, appeal to physical, visual, and geometric intuition.



                                  For applications, tell your audience that topology is a great reduction tool in many problems that at first seem complicated, examples being persistent homology, Morse theory, and topological degree theory. The IMA is having a special focus year on applications of algebraic topology (http://www.ima.umn.edu/2013-2014/) - check out the lectures! And make special mention of the computability of the theory - homology is a great tool because we can make the computer do it.



                                  Edit: I was speaking primarily about explaining topology to a general audience. I don't know much about homotopy groups, but if I were to say anything about fundamental groups I would make sure to mention the Poincare conjecture. It wouldn't hurt to mention how it had a USD 1 million bounty attached to it, and besides attempts at Poincare led us to some of the most fruitful mathematics of the past century, such as Morse theory.







                                  share|cite|improve this answer














                                  share|cite|improve this answer



                                  share|cite|improve this answer








                                  edited Nov 4 '13 at 11:43









                                  azimut

                                  16.6k1052102




                                  16.6k1052102










                                  answered Oct 25 '13 at 22:58









                                  Gyu Eun LeeGyu Eun Lee

                                  13.4k2354




                                  13.4k2354























                                      7












                                      $begingroup$

                                      You might want to watch the talk of Tim Gowers "The Importance of Mathematics".



                                      In an nutshell, his answer to the politician was that some areas of mathematics have more practical applications than others but:




                                      1. It is very hard to predict which areas of mathematics will have applications in the future.

                                      2. The different areas of mathematics are very interconnected. Cutting the funding of a "non-profitable" area could harm the development of "profitable" ones.


                                      Of course he also talked about beauty and the quest for knowledge.






                                      share|cite|improve this answer









                                      $endgroup$


















                                        7












                                        $begingroup$

                                        You might want to watch the talk of Tim Gowers "The Importance of Mathematics".



                                        In an nutshell, his answer to the politician was that some areas of mathematics have more practical applications than others but:




                                        1. It is very hard to predict which areas of mathematics will have applications in the future.

                                        2. The different areas of mathematics are very interconnected. Cutting the funding of a "non-profitable" area could harm the development of "profitable" ones.


                                        Of course he also talked about beauty and the quest for knowledge.






                                        share|cite|improve this answer









                                        $endgroup$
















                                          7












                                          7








                                          7





                                          $begingroup$

                                          You might want to watch the talk of Tim Gowers "The Importance of Mathematics".



                                          In an nutshell, his answer to the politician was that some areas of mathematics have more practical applications than others but:




                                          1. It is very hard to predict which areas of mathematics will have applications in the future.

                                          2. The different areas of mathematics are very interconnected. Cutting the funding of a "non-profitable" area could harm the development of "profitable" ones.


                                          Of course he also talked about beauty and the quest for knowledge.






                                          share|cite|improve this answer









                                          $endgroup$



                                          You might want to watch the talk of Tim Gowers "The Importance of Mathematics".



                                          In an nutshell, his answer to the politician was that some areas of mathematics have more practical applications than others but:




                                          1. It is very hard to predict which areas of mathematics will have applications in the future.

                                          2. The different areas of mathematics are very interconnected. Cutting the funding of a "non-profitable" area could harm the development of "profitable" ones.


                                          Of course he also talked about beauty and the quest for knowledge.







                                          share|cite|improve this answer












                                          share|cite|improve this answer



                                          share|cite|improve this answer










                                          answered Oct 26 '13 at 1:30









                                          posilonposilon

                                          1,470516




                                          1,470516























                                              7












                                              $begingroup$

                                              I belive that you can find really interesting this stuff posted by Prof. Porter. It's not about the foundamental group but nonetheless present some real world application in algebraic topology (and there are some links to some documents too).



                                              That is an answer to a post of Tom Leinster addressing a similar question: he was looking for application of abstract math to motivate applied mathematicians into studying abstract maths and the opposite, find interesting application that motivates abstract maths for theoretical mathematicians.
                                              here the complete link.



                                              Anyway if you try to google a little bit you can find a lot of application of algebraic topology in applied field, in particular in data analysis and machine learning. If I'm not mistaken there are lots of work in persistent homology that have real world application. Unfortunately I don't have papers under my hand but try to look you'll most certain find lots of stuff.



                                              Now you could argue that these stuff doesn't directly regard fundamental group
                                              none the less to study such object it comes in handy to know a lot about fundamental groups. That's more lots of this applied tools were born in order to solve theoretical problems and later it was discovered they could be used into real world application. They are a sort of practice proof that theoretical research is as much important as applied one.



                                              Hope this very short and poor answer could help.
                                              Have good luck with you work :)



                                              Edit: After a while a was able to retrieve this old post on MathOverflow which I believe can be useful. There are some real world application of algebraic topology with some reference too.






                                              share|cite|improve this answer











                                              $endgroup$


















                                                7












                                                $begingroup$

                                                I belive that you can find really interesting this stuff posted by Prof. Porter. It's not about the foundamental group but nonetheless present some real world application in algebraic topology (and there are some links to some documents too).



                                                That is an answer to a post of Tom Leinster addressing a similar question: he was looking for application of abstract math to motivate applied mathematicians into studying abstract maths and the opposite, find interesting application that motivates abstract maths for theoretical mathematicians.
                                                here the complete link.



                                                Anyway if you try to google a little bit you can find a lot of application of algebraic topology in applied field, in particular in data analysis and machine learning. If I'm not mistaken there are lots of work in persistent homology that have real world application. Unfortunately I don't have papers under my hand but try to look you'll most certain find lots of stuff.



                                                Now you could argue that these stuff doesn't directly regard fundamental group
                                                none the less to study such object it comes in handy to know a lot about fundamental groups. That's more lots of this applied tools were born in order to solve theoretical problems and later it was discovered they could be used into real world application. They are a sort of practice proof that theoretical research is as much important as applied one.



                                                Hope this very short and poor answer could help.
                                                Have good luck with you work :)



                                                Edit: After a while a was able to retrieve this old post on MathOverflow which I believe can be useful. There are some real world application of algebraic topology with some reference too.






                                                share|cite|improve this answer











                                                $endgroup$
















                                                  7












                                                  7








                                                  7





                                                  $begingroup$

                                                  I belive that you can find really interesting this stuff posted by Prof. Porter. It's not about the foundamental group but nonetheless present some real world application in algebraic topology (and there are some links to some documents too).



                                                  That is an answer to a post of Tom Leinster addressing a similar question: he was looking for application of abstract math to motivate applied mathematicians into studying abstract maths and the opposite, find interesting application that motivates abstract maths for theoretical mathematicians.
                                                  here the complete link.



                                                  Anyway if you try to google a little bit you can find a lot of application of algebraic topology in applied field, in particular in data analysis and machine learning. If I'm not mistaken there are lots of work in persistent homology that have real world application. Unfortunately I don't have papers under my hand but try to look you'll most certain find lots of stuff.



                                                  Now you could argue that these stuff doesn't directly regard fundamental group
                                                  none the less to study such object it comes in handy to know a lot about fundamental groups. That's more lots of this applied tools were born in order to solve theoretical problems and later it was discovered they could be used into real world application. They are a sort of practice proof that theoretical research is as much important as applied one.



                                                  Hope this very short and poor answer could help.
                                                  Have good luck with you work :)



                                                  Edit: After a while a was able to retrieve this old post on MathOverflow which I believe can be useful. There are some real world application of algebraic topology with some reference too.






                                                  share|cite|improve this answer











                                                  $endgroup$



                                                  I belive that you can find really interesting this stuff posted by Prof. Porter. It's not about the foundamental group but nonetheless present some real world application in algebraic topology (and there are some links to some documents too).



                                                  That is an answer to a post of Tom Leinster addressing a similar question: he was looking for application of abstract math to motivate applied mathematicians into studying abstract maths and the opposite, find interesting application that motivates abstract maths for theoretical mathematicians.
                                                  here the complete link.



                                                  Anyway if you try to google a little bit you can find a lot of application of algebraic topology in applied field, in particular in data analysis and machine learning. If I'm not mistaken there are lots of work in persistent homology that have real world application. Unfortunately I don't have papers under my hand but try to look you'll most certain find lots of stuff.



                                                  Now you could argue that these stuff doesn't directly regard fundamental group
                                                  none the less to study such object it comes in handy to know a lot about fundamental groups. That's more lots of this applied tools were born in order to solve theoretical problems and later it was discovered they could be used into real world application. They are a sort of practice proof that theoretical research is as much important as applied one.



                                                  Hope this very short and poor answer could help.
                                                  Have good luck with you work :)



                                                  Edit: After a while a was able to retrieve this old post on MathOverflow which I believe can be useful. There are some real world application of algebraic topology with some reference too.







                                                  share|cite|improve this answer














                                                  share|cite|improve this answer



                                                  share|cite|improve this answer








                                                  edited Apr 13 '17 at 12:58









                                                  Community

                                                  1




                                                  1










                                                  answered Oct 25 '13 at 21:29









                                                  Giorgio MossaGiorgio Mossa

                                                  14.4k11750




                                                  14.4k11750























                                                      5












                                                      $begingroup$

                                                      You might consider looking into topological mixing. One studies the optimal way you can mix, for instance, a viscous fluid with n-rods. This applies to such things as taffy pullers. Here are slides from a professor at my institution.






                                                      share|cite|improve this answer









                                                      $endgroup$


















                                                        5












                                                        $begingroup$

                                                        You might consider looking into topological mixing. One studies the optimal way you can mix, for instance, a viscous fluid with n-rods. This applies to such things as taffy pullers. Here are slides from a professor at my institution.






                                                        share|cite|improve this answer









                                                        $endgroup$
















                                                          5












                                                          5








                                                          5





                                                          $begingroup$

                                                          You might consider looking into topological mixing. One studies the optimal way you can mix, for instance, a viscous fluid with n-rods. This applies to such things as taffy pullers. Here are slides from a professor at my institution.






                                                          share|cite|improve this answer









                                                          $endgroup$



                                                          You might consider looking into topological mixing. One studies the optimal way you can mix, for instance, a viscous fluid with n-rods. This applies to such things as taffy pullers. Here are slides from a professor at my institution.







                                                          share|cite|improve this answer












                                                          share|cite|improve this answer



                                                          share|cite|improve this answer










                                                          answered Oct 26 '13 at 0:25









                                                          abnryabnry

                                                          11.7k22367




                                                          11.7k22367























                                                              5












                                                              $begingroup$

                                                              Here is something that may be close to the politically minded hearts - the Human Heart! I have heard that methods of algebraic topology have been used in medical research of the human heart.




                                                              1. When I was in grad school (late 80s), there was some documentary (may be just a news clip?) about them meds diagnosing imminent heart problems from the topology of the electric currents on the heart tissue. On the chalkboard behind the docs the calculations definitely included homotopy groups - don't remember whether it was the fundamental group. This was probably related to the knots formed by those currents, but obviously they didn't give the details. But something like: "the 2-torsion of the $n$th homotopy group of your heart currents changes => you will soon develop arrythmia (I'm making this up) would sound kinda convincing.

                                                              2. When the technology of 3D-imaging of the human body was emerging, the people at the general hospital across the street from the sciences building asked a colleague the following question (he had helped them with compression algorithms and other handling of medical images before, so became the obvious target): the computer has stored a 3D-image of the heart of the patient as a 3D-array of small cubes, and can reliably tell whether each cube contains heart tissue or is empty (like part of an atrium or a ventricle). If that data is presented in a useful 3D-image, a trained human can quickly spot whether there are extra holes in the walls separating the chambers. But can this process be automated? We talked it over a lunch, and the best suggestion we came up with was to calculate the Euler characteristic of the 2D-surface formed by the boundary of the chambers (connectedness also). Surely the same solution had been proposed earlier elsewhere, but I think it was adopted (though there may be difficult to analyze border effects due to the granularity of the 3D-image). Anyway: letting the computer calculate the homology groups of the heart => the docs know whether heart surgery is needed doesn't sound too useless an application.


                                                              Keeping this CW, because this is largely vague recollections, second (or third) hand information and such. Welcoming edits/links to references.






                                                              share|cite|improve this answer











                                                              $endgroup$









                                                              • 2




                                                                $begingroup$
                                                                I remembered having read something about heart failure and topology a long time ago in the Scientific American and found this link.
                                                                $endgroup$
                                                                – WimC
                                                                Oct 26 '13 at 19:17
















                                                              5












                                                              $begingroup$

                                                              Here is something that may be close to the politically minded hearts - the Human Heart! I have heard that methods of algebraic topology have been used in medical research of the human heart.




                                                              1. When I was in grad school (late 80s), there was some documentary (may be just a news clip?) about them meds diagnosing imminent heart problems from the topology of the electric currents on the heart tissue. On the chalkboard behind the docs the calculations definitely included homotopy groups - don't remember whether it was the fundamental group. This was probably related to the knots formed by those currents, but obviously they didn't give the details. But something like: "the 2-torsion of the $n$th homotopy group of your heart currents changes => you will soon develop arrythmia (I'm making this up) would sound kinda convincing.

                                                              2. When the technology of 3D-imaging of the human body was emerging, the people at the general hospital across the street from the sciences building asked a colleague the following question (he had helped them with compression algorithms and other handling of medical images before, so became the obvious target): the computer has stored a 3D-image of the heart of the patient as a 3D-array of small cubes, and can reliably tell whether each cube contains heart tissue or is empty (like part of an atrium or a ventricle). If that data is presented in a useful 3D-image, a trained human can quickly spot whether there are extra holes in the walls separating the chambers. But can this process be automated? We talked it over a lunch, and the best suggestion we came up with was to calculate the Euler characteristic of the 2D-surface formed by the boundary of the chambers (connectedness also). Surely the same solution had been proposed earlier elsewhere, but I think it was adopted (though there may be difficult to analyze border effects due to the granularity of the 3D-image). Anyway: letting the computer calculate the homology groups of the heart => the docs know whether heart surgery is needed doesn't sound too useless an application.


                                                              Keeping this CW, because this is largely vague recollections, second (or third) hand information and such. Welcoming edits/links to references.






                                                              share|cite|improve this answer











                                                              $endgroup$









                                                              • 2




                                                                $begingroup$
                                                                I remembered having read something about heart failure and topology a long time ago in the Scientific American and found this link.
                                                                $endgroup$
                                                                – WimC
                                                                Oct 26 '13 at 19:17














                                                              5












                                                              5








                                                              5





                                                              $begingroup$

                                                              Here is something that may be close to the politically minded hearts - the Human Heart! I have heard that methods of algebraic topology have been used in medical research of the human heart.




                                                              1. When I was in grad school (late 80s), there was some documentary (may be just a news clip?) about them meds diagnosing imminent heart problems from the topology of the electric currents on the heart tissue. On the chalkboard behind the docs the calculations definitely included homotopy groups - don't remember whether it was the fundamental group. This was probably related to the knots formed by those currents, but obviously they didn't give the details. But something like: "the 2-torsion of the $n$th homotopy group of your heart currents changes => you will soon develop arrythmia (I'm making this up) would sound kinda convincing.

                                                              2. When the technology of 3D-imaging of the human body was emerging, the people at the general hospital across the street from the sciences building asked a colleague the following question (he had helped them with compression algorithms and other handling of medical images before, so became the obvious target): the computer has stored a 3D-image of the heart of the patient as a 3D-array of small cubes, and can reliably tell whether each cube contains heart tissue or is empty (like part of an atrium or a ventricle). If that data is presented in a useful 3D-image, a trained human can quickly spot whether there are extra holes in the walls separating the chambers. But can this process be automated? We talked it over a lunch, and the best suggestion we came up with was to calculate the Euler characteristic of the 2D-surface formed by the boundary of the chambers (connectedness also). Surely the same solution had been proposed earlier elsewhere, but I think it was adopted (though there may be difficult to analyze border effects due to the granularity of the 3D-image). Anyway: letting the computer calculate the homology groups of the heart => the docs know whether heart surgery is needed doesn't sound too useless an application.


                                                              Keeping this CW, because this is largely vague recollections, second (or third) hand information and such. Welcoming edits/links to references.






                                                              share|cite|improve this answer











                                                              $endgroup$



                                                              Here is something that may be close to the politically minded hearts - the Human Heart! I have heard that methods of algebraic topology have been used in medical research of the human heart.




                                                              1. When I was in grad school (late 80s), there was some documentary (may be just a news clip?) about them meds diagnosing imminent heart problems from the topology of the electric currents on the heart tissue. On the chalkboard behind the docs the calculations definitely included homotopy groups - don't remember whether it was the fundamental group. This was probably related to the knots formed by those currents, but obviously they didn't give the details. But something like: "the 2-torsion of the $n$th homotopy group of your heart currents changes => you will soon develop arrythmia (I'm making this up) would sound kinda convincing.

                                                              2. When the technology of 3D-imaging of the human body was emerging, the people at the general hospital across the street from the sciences building asked a colleague the following question (he had helped them with compression algorithms and other handling of medical images before, so became the obvious target): the computer has stored a 3D-image of the heart of the patient as a 3D-array of small cubes, and can reliably tell whether each cube contains heart tissue or is empty (like part of an atrium or a ventricle). If that data is presented in a useful 3D-image, a trained human can quickly spot whether there are extra holes in the walls separating the chambers. But can this process be automated? We talked it over a lunch, and the best suggestion we came up with was to calculate the Euler characteristic of the 2D-surface formed by the boundary of the chambers (connectedness also). Surely the same solution had been proposed earlier elsewhere, but I think it was adopted (though there may be difficult to analyze border effects due to the granularity of the 3D-image). Anyway: letting the computer calculate the homology groups of the heart => the docs know whether heart surgery is needed doesn't sound too useless an application.


                                                              Keeping this CW, because this is largely vague recollections, second (or third) hand information and such. Welcoming edits/links to references.







                                                              share|cite|improve this answer














                                                              share|cite|improve this answer



                                                              share|cite|improve this answer








                                                              answered Oct 26 '13 at 7:34


























                                                              community wiki





                                                              Jyrki Lahtonen









                                                              • 2




                                                                $begingroup$
                                                                I remembered having read something about heart failure and topology a long time ago in the Scientific American and found this link.
                                                                $endgroup$
                                                                – WimC
                                                                Oct 26 '13 at 19:17














                                                              • 2




                                                                $begingroup$
                                                                I remembered having read something about heart failure and topology a long time ago in the Scientific American and found this link.
                                                                $endgroup$
                                                                – WimC
                                                                Oct 26 '13 at 19:17








                                                              2




                                                              2




                                                              $begingroup$
                                                              I remembered having read something about heart failure and topology a long time ago in the Scientific American and found this link.
                                                              $endgroup$
                                                              – WimC
                                                              Oct 26 '13 at 19:17




                                                              $begingroup$
                                                              I remembered having read something about heart failure and topology a long time ago in the Scientific American and found this link.
                                                              $endgroup$
                                                              – WimC
                                                              Oct 26 '13 at 19:17











                                                              5












                                                              $begingroup$

                                                              I worked in the classification of 3-manifolds. Briefly, here's what I came up with to explain this to people who were really interested. This doesn't so much justify the work politically, but it does give the general idea.




                                                              • Imagine you're a two-dimensional creature. What could your world look like? It could be a plane. It could also be a sphere, which would make your life different because you always come back where you started. Or it could be the surface of a doughnut, which would be different again: there are two ways to loop around, and you know that they're different.
                                                                The size of the sphere or doughnut doesn't matter, just the overall shape. The universe in the video game Asteroids is also a doughnut. This is "topology".


                                                              • In fact there's an infinite list of universes: the plane, the sphere, the doughnut, doughnuts with any number of holes, some others which are twisted like a Moebius strip...


                                                              • People solved this problem in the 19th century: we have a complete list of the two-dimensional universes, and if you give a two-dimensional universe, I can easily tell you which one it is. We have a "classification". The main tool for this classification is to use numbers to describe the shapes. That's "algebraic topology".


                                                              • We seem to be living in a three-dimensional universe. What shape is it? What shapes could it be? Cosmologists, for example, would need to know this. But it turns out to be a much harder classification problem, and only partly solved. I worked on tools of algebraic topology to help with this problem.







                                                              share|cite|improve this answer









                                                              $endgroup$


















                                                                5












                                                                $begingroup$

                                                                I worked in the classification of 3-manifolds. Briefly, here's what I came up with to explain this to people who were really interested. This doesn't so much justify the work politically, but it does give the general idea.




                                                                • Imagine you're a two-dimensional creature. What could your world look like? It could be a plane. It could also be a sphere, which would make your life different because you always come back where you started. Or it could be the surface of a doughnut, which would be different again: there are two ways to loop around, and you know that they're different.
                                                                  The size of the sphere or doughnut doesn't matter, just the overall shape. The universe in the video game Asteroids is also a doughnut. This is "topology".


                                                                • In fact there's an infinite list of universes: the plane, the sphere, the doughnut, doughnuts with any number of holes, some others which are twisted like a Moebius strip...


                                                                • People solved this problem in the 19th century: we have a complete list of the two-dimensional universes, and if you give a two-dimensional universe, I can easily tell you which one it is. We have a "classification". The main tool for this classification is to use numbers to describe the shapes. That's "algebraic topology".


                                                                • We seem to be living in a three-dimensional universe. What shape is it? What shapes could it be? Cosmologists, for example, would need to know this. But it turns out to be a much harder classification problem, and only partly solved. I worked on tools of algebraic topology to help with this problem.







                                                                share|cite|improve this answer









                                                                $endgroup$
















                                                                  5












                                                                  5








                                                                  5





                                                                  $begingroup$

                                                                  I worked in the classification of 3-manifolds. Briefly, here's what I came up with to explain this to people who were really interested. This doesn't so much justify the work politically, but it does give the general idea.




                                                                  • Imagine you're a two-dimensional creature. What could your world look like? It could be a plane. It could also be a sphere, which would make your life different because you always come back where you started. Or it could be the surface of a doughnut, which would be different again: there are two ways to loop around, and you know that they're different.
                                                                    The size of the sphere or doughnut doesn't matter, just the overall shape. The universe in the video game Asteroids is also a doughnut. This is "topology".


                                                                  • In fact there's an infinite list of universes: the plane, the sphere, the doughnut, doughnuts with any number of holes, some others which are twisted like a Moebius strip...


                                                                  • People solved this problem in the 19th century: we have a complete list of the two-dimensional universes, and if you give a two-dimensional universe, I can easily tell you which one it is. We have a "classification". The main tool for this classification is to use numbers to describe the shapes. That's "algebraic topology".


                                                                  • We seem to be living in a three-dimensional universe. What shape is it? What shapes could it be? Cosmologists, for example, would need to know this. But it turns out to be a much harder classification problem, and only partly solved. I worked on tools of algebraic topology to help with this problem.







                                                                  share|cite|improve this answer









                                                                  $endgroup$



                                                                  I worked in the classification of 3-manifolds. Briefly, here's what I came up with to explain this to people who were really interested. This doesn't so much justify the work politically, but it does give the general idea.




                                                                  • Imagine you're a two-dimensional creature. What could your world look like? It could be a plane. It could also be a sphere, which would make your life different because you always come back where you started. Or it could be the surface of a doughnut, which would be different again: there are two ways to loop around, and you know that they're different.
                                                                    The size of the sphere or doughnut doesn't matter, just the overall shape. The universe in the video game Asteroids is also a doughnut. This is "topology".


                                                                  • In fact there's an infinite list of universes: the plane, the sphere, the doughnut, doughnuts with any number of holes, some others which are twisted like a Moebius strip...


                                                                  • People solved this problem in the 19th century: we have a complete list of the two-dimensional universes, and if you give a two-dimensional universe, I can easily tell you which one it is. We have a "classification". The main tool for this classification is to use numbers to describe the shapes. That's "algebraic topology".


                                                                  • We seem to be living in a three-dimensional universe. What shape is it? What shapes could it be? Cosmologists, for example, would need to know this. But it turns out to be a much harder classification problem, and only partly solved. I worked on tools of algebraic topology to help with this problem.








                                                                  share|cite|improve this answer












                                                                  share|cite|improve this answer



                                                                  share|cite|improve this answer










                                                                  answered Oct 26 '13 at 14:14









                                                                  Hew WolffHew Wolff

                                                                  2,270717




                                                                  2,270717























                                                                      4












                                                                      $begingroup$

                                                                      Have a look at the Knot Exhibition on this website and the applications there, including the history of knots, which seems to date back at least 300 millenia. One scientific application is to Knots and DNA, you can also do a web search on this. The design of the exhibition is to show how the work of understanding knots and their mathematics comes before you can make the applications. Tim Porter and I also did an article on Making a mathematical exhibition. See also Out of Line.



                                                                      The following picture shows a loop of string tied around part of a pentoil knot according to the formula



                                                                      $$xyxyxy^{-1} x^{-1} y^{-1} x^{-1} y^{-1} $$



                                                                      pent



                                                                      Then the loop of string comes off the knot! Explanation is in this seminar at Liverpool, for a demonstration this with a well travelled pentoil knot made of copper tubing and a length of thin rope.



                                                                      The idea for this trick comes from the determination of the fundamental group of the complement of the pentoil knot in $3$-space. See also pages 350-351 of Topology and Groupoids.






                                                                      share|cite|improve this answer











                                                                      $endgroup$


















                                                                        4












                                                                        $begingroup$

                                                                        Have a look at the Knot Exhibition on this website and the applications there, including the history of knots, which seems to date back at least 300 millenia. One scientific application is to Knots and DNA, you can also do a web search on this. The design of the exhibition is to show how the work of understanding knots and their mathematics comes before you can make the applications. Tim Porter and I also did an article on Making a mathematical exhibition. See also Out of Line.



                                                                        The following picture shows a loop of string tied around part of a pentoil knot according to the formula



                                                                        $$xyxyxy^{-1} x^{-1} y^{-1} x^{-1} y^{-1} $$



                                                                        pent



                                                                        Then the loop of string comes off the knot! Explanation is in this seminar at Liverpool, for a demonstration this with a well travelled pentoil knot made of copper tubing and a length of thin rope.



                                                                        The idea for this trick comes from the determination of the fundamental group of the complement of the pentoil knot in $3$-space. See also pages 350-351 of Topology and Groupoids.






                                                                        share|cite|improve this answer











                                                                        $endgroup$
















                                                                          4












                                                                          4








                                                                          4





                                                                          $begingroup$

                                                                          Have a look at the Knot Exhibition on this website and the applications there, including the history of knots, which seems to date back at least 300 millenia. One scientific application is to Knots and DNA, you can also do a web search on this. The design of the exhibition is to show how the work of understanding knots and their mathematics comes before you can make the applications. Tim Porter and I also did an article on Making a mathematical exhibition. See also Out of Line.



                                                                          The following picture shows a loop of string tied around part of a pentoil knot according to the formula



                                                                          $$xyxyxy^{-1} x^{-1} y^{-1} x^{-1} y^{-1} $$



                                                                          pent



                                                                          Then the loop of string comes off the knot! Explanation is in this seminar at Liverpool, for a demonstration this with a well travelled pentoil knot made of copper tubing and a length of thin rope.



                                                                          The idea for this trick comes from the determination of the fundamental group of the complement of the pentoil knot in $3$-space. See also pages 350-351 of Topology and Groupoids.






                                                                          share|cite|improve this answer











                                                                          $endgroup$



                                                                          Have a look at the Knot Exhibition on this website and the applications there, including the history of knots, which seems to date back at least 300 millenia. One scientific application is to Knots and DNA, you can also do a web search on this. The design of the exhibition is to show how the work of understanding knots and their mathematics comes before you can make the applications. Tim Porter and I also did an article on Making a mathematical exhibition. See also Out of Line.



                                                                          The following picture shows a loop of string tied around part of a pentoil knot according to the formula



                                                                          $$xyxyxy^{-1} x^{-1} y^{-1} x^{-1} y^{-1} $$



                                                                          pent



                                                                          Then the loop of string comes off the knot! Explanation is in this seminar at Liverpool, for a demonstration this with a well travelled pentoil knot made of copper tubing and a length of thin rope.



                                                                          The idea for this trick comes from the determination of the fundamental group of the complement of the pentoil knot in $3$-space. See also pages 350-351 of Topology and Groupoids.







                                                                          share|cite|improve this answer














                                                                          share|cite|improve this answer



                                                                          share|cite|improve this answer








                                                                          edited Jan 25 at 22:05

























                                                                          answered Nov 6 '13 at 22:15









                                                                          Ronnie BrownRonnie Brown

                                                                          12.2k13039




                                                                          12.2k13039























                                                                              3












                                                                              $begingroup$

                                                                              I like the pictures from chapter 0 of Hatcher for that purpose.



                                                                              hatcher chapter 0



                                                                              And also Louis Kauffman made some nice pictures for the Fox calculus.



                                                                              fox calculus



                                                                              And Thurston's 3-manifolds book has good pictures as well.
                                                                              thurston



                                                                              "Normal maths works with squares, triangles, perfect circles; it counts beans. In topology we're freer to work with more interesting shapes." (cue picture from Dr Seuss)



                                                                              seuss



                                                                              This is basically what I did in http://tmblr.co/ZdCxIydt_Owi, drawing on the familiar philosophical distinction between reductivism vs holism to work in a cohomology element (as an argument for holism: some things can't be decomposed into atoms without losing some essential property that only shows up in how they're put together).



                                                                              A romantic notion I felt when I was first reading Hatcher is that the "counting rocks" and "straight-sight cannonball" problems that seem more typical of a more boring mathematics, get replaced by "feminine" "weaving-based" mathematics. (Women were the ones tasked with purling, picking, and wrapping in most of history.) So this is a nice tie-in for non-mathematicians, although I'm not sure AT actually is more "feminine" (whatever that means) it's an attention-grabber.





                                                                              You can also talk about topological spaces vs metric spaces by using highways



                                                                              USA road network



                                                                              versus airplane connections



                                                                              paris air connection



                                                                              taking a ring road versus trying to not back-track, versus taking small streets with different speed limits, could be thought of as metrics -- whereas if you're at the airport, then it's a question more of does the city you're in actually connect to where you're going or not, rather than 'physical' distance. You might end up flying past your destination (or at least extra distance) before taking a second flight/network hop, so the "network distance" is two (2 nbds), compare that to "ground distance". (Maybe for example Colorado Springs does not receive flights from L.A., so one has to go further east to Denver first ... if my geography is wrong then some pattern like what I'm suggesting does exist somewhere.)





                                                                              As far as applications, I like the suggestions of Ghrist & Carlsson's work. I would add Ghrist's robots-not-bumping-into-each-other in warehouses, anyone can easily envisage as a braid the robots not bumping into each other over time. (See Ghrist's EAT book ch 1 for the relevant picture.)



                                                                              ghrist braids robot motion planning factory floor



                                                                              But I would make a further point, which is that the goal is not to rehash the applications already developed, but rather why the novel approaches might lead to something new. In other words, "How has this already been applied?" and "How might this generate applications in the future?" are two separate questions.



                                                                              To that end I think it's fine, as long as you use clear non-technical language, to talk in generalities about how the "style" of topological thinking differs from maths they may be more familiar with. You could give some examples of "wiggly" or "non-rigid" situations and say that the ambition is that with enough development of AT, these situations could be handled.



                                                                              Justin Curry made a pitch like this in http://arxiv.org/abs/1303.3255, saying essentially "Linear algebra was amazing for applications; I think my thing could be similarly productive, from a different starting point."



                                                                              On the other hand, we probably all know examples of scientists who thought their research would be used for totally different purposes than what it ended up in. My favourite example is the inventor of NMR (worth a Nobel prize) thought NMR would be used for "calibrating industrial magnets". Instead it was used to look inside people's brains. (But that was someone else's work, to apply it to that.)



                                                                              On the other other hand, probably no politicians slapped Purcell & Bloch on the wrist when they said they were working on technology to calibrate industrial magnets. That can be envisaged by a politician and sounds science-y and maybe-important (just not as important as what it ended up being). Some mathematicians have also been known to name their ideas for marketing purposes, for example "dynamic programming" and "information entropy". (There are funny stories going along with the naming of each of these.)





                                                                              If I had a piece of paper, my 10-second elevator pitch for algebraic topology would be:




                                                                              1. Most data-analysis methods assume data is "Square" (meaning Euclidean--independent dimensions, flat, etc)

                                                                              2. But how do we know that? What if the data is shaped like a peace sign? pax-benz

                                                                              3. We don't want to be caught off-guard by assumptions we weren't necessarily trying to make, so do AT to be aware of other possibilities the world might hit you with.


                                                                              thor vigfursson






                                                                              share|cite|improve this answer











                                                                              $endgroup$


















                                                                                3












                                                                                $begingroup$

                                                                                I like the pictures from chapter 0 of Hatcher for that purpose.



                                                                                hatcher chapter 0



                                                                                And also Louis Kauffman made some nice pictures for the Fox calculus.



                                                                                fox calculus



                                                                                And Thurston's 3-manifolds book has good pictures as well.
                                                                                thurston



                                                                                "Normal maths works with squares, triangles, perfect circles; it counts beans. In topology we're freer to work with more interesting shapes." (cue picture from Dr Seuss)



                                                                                seuss



                                                                                This is basically what I did in http://tmblr.co/ZdCxIydt_Owi, drawing on the familiar philosophical distinction between reductivism vs holism to work in a cohomology element (as an argument for holism: some things can't be decomposed into atoms without losing some essential property that only shows up in how they're put together).



                                                                                A romantic notion I felt when I was first reading Hatcher is that the "counting rocks" and "straight-sight cannonball" problems that seem more typical of a more boring mathematics, get replaced by "feminine" "weaving-based" mathematics. (Women were the ones tasked with purling, picking, and wrapping in most of history.) So this is a nice tie-in for non-mathematicians, although I'm not sure AT actually is more "feminine" (whatever that means) it's an attention-grabber.





                                                                                You can also talk about topological spaces vs metric spaces by using highways



                                                                                USA road network



                                                                                versus airplane connections



                                                                                paris air connection



                                                                                taking a ring road versus trying to not back-track, versus taking small streets with different speed limits, could be thought of as metrics -- whereas if you're at the airport, then it's a question more of does the city you're in actually connect to where you're going or not, rather than 'physical' distance. You might end up flying past your destination (or at least extra distance) before taking a second flight/network hop, so the "network distance" is two (2 nbds), compare that to "ground distance". (Maybe for example Colorado Springs does not receive flights from L.A., so one has to go further east to Denver first ... if my geography is wrong then some pattern like what I'm suggesting does exist somewhere.)





                                                                                As far as applications, I like the suggestions of Ghrist & Carlsson's work. I would add Ghrist's robots-not-bumping-into-each-other in warehouses, anyone can easily envisage as a braid the robots not bumping into each other over time. (See Ghrist's EAT book ch 1 for the relevant picture.)



                                                                                ghrist braids robot motion planning factory floor



                                                                                But I would make a further point, which is that the goal is not to rehash the applications already developed, but rather why the novel approaches might lead to something new. In other words, "How has this already been applied?" and "How might this generate applications in the future?" are two separate questions.



                                                                                To that end I think it's fine, as long as you use clear non-technical language, to talk in generalities about how the "style" of topological thinking differs from maths they may be more familiar with. You could give some examples of "wiggly" or "non-rigid" situations and say that the ambition is that with enough development of AT, these situations could be handled.



                                                                                Justin Curry made a pitch like this in http://arxiv.org/abs/1303.3255, saying essentially "Linear algebra was amazing for applications; I think my thing could be similarly productive, from a different starting point."



                                                                                On the other hand, we probably all know examples of scientists who thought their research would be used for totally different purposes than what it ended up in. My favourite example is the inventor of NMR (worth a Nobel prize) thought NMR would be used for "calibrating industrial magnets". Instead it was used to look inside people's brains. (But that was someone else's work, to apply it to that.)



                                                                                On the other other hand, probably no politicians slapped Purcell & Bloch on the wrist when they said they were working on technology to calibrate industrial magnets. That can be envisaged by a politician and sounds science-y and maybe-important (just not as important as what it ended up being). Some mathematicians have also been known to name their ideas for marketing purposes, for example "dynamic programming" and "information entropy". (There are funny stories going along with the naming of each of these.)





                                                                                If I had a piece of paper, my 10-second elevator pitch for algebraic topology would be:




                                                                                1. Most data-analysis methods assume data is "Square" (meaning Euclidean--independent dimensions, flat, etc)

                                                                                2. But how do we know that? What if the data is shaped like a peace sign? pax-benz

                                                                                3. We don't want to be caught off-guard by assumptions we weren't necessarily trying to make, so do AT to be aware of other possibilities the world might hit you with.


                                                                                thor vigfursson






                                                                                share|cite|improve this answer











                                                                                $endgroup$
















                                                                                  3












                                                                                  3








                                                                                  3





                                                                                  $begingroup$

                                                                                  I like the pictures from chapter 0 of Hatcher for that purpose.



                                                                                  hatcher chapter 0



                                                                                  And also Louis Kauffman made some nice pictures for the Fox calculus.



                                                                                  fox calculus



                                                                                  And Thurston's 3-manifolds book has good pictures as well.
                                                                                  thurston



                                                                                  "Normal maths works with squares, triangles, perfect circles; it counts beans. In topology we're freer to work with more interesting shapes." (cue picture from Dr Seuss)



                                                                                  seuss



                                                                                  This is basically what I did in http://tmblr.co/ZdCxIydt_Owi, drawing on the familiar philosophical distinction between reductivism vs holism to work in a cohomology element (as an argument for holism: some things can't be decomposed into atoms without losing some essential property that only shows up in how they're put together).



                                                                                  A romantic notion I felt when I was first reading Hatcher is that the "counting rocks" and "straight-sight cannonball" problems that seem more typical of a more boring mathematics, get replaced by "feminine" "weaving-based" mathematics. (Women were the ones tasked with purling, picking, and wrapping in most of history.) So this is a nice tie-in for non-mathematicians, although I'm not sure AT actually is more "feminine" (whatever that means) it's an attention-grabber.





                                                                                  You can also talk about topological spaces vs metric spaces by using highways



                                                                                  USA road network



                                                                                  versus airplane connections



                                                                                  paris air connection



                                                                                  taking a ring road versus trying to not back-track, versus taking small streets with different speed limits, could be thought of as metrics -- whereas if you're at the airport, then it's a question more of does the city you're in actually connect to where you're going or not, rather than 'physical' distance. You might end up flying past your destination (or at least extra distance) before taking a second flight/network hop, so the "network distance" is two (2 nbds), compare that to "ground distance". (Maybe for example Colorado Springs does not receive flights from L.A., so one has to go further east to Denver first ... if my geography is wrong then some pattern like what I'm suggesting does exist somewhere.)





                                                                                  As far as applications, I like the suggestions of Ghrist & Carlsson's work. I would add Ghrist's robots-not-bumping-into-each-other in warehouses, anyone can easily envisage as a braid the robots not bumping into each other over time. (See Ghrist's EAT book ch 1 for the relevant picture.)



                                                                                  ghrist braids robot motion planning factory floor



                                                                                  But I would make a further point, which is that the goal is not to rehash the applications already developed, but rather why the novel approaches might lead to something new. In other words, "How has this already been applied?" and "How might this generate applications in the future?" are two separate questions.



                                                                                  To that end I think it's fine, as long as you use clear non-technical language, to talk in generalities about how the "style" of topological thinking differs from maths they may be more familiar with. You could give some examples of "wiggly" or "non-rigid" situations and say that the ambition is that with enough development of AT, these situations could be handled.



                                                                                  Justin Curry made a pitch like this in http://arxiv.org/abs/1303.3255, saying essentially "Linear algebra was amazing for applications; I think my thing could be similarly productive, from a different starting point."



                                                                                  On the other hand, we probably all know examples of scientists who thought their research would be used for totally different purposes than what it ended up in. My favourite example is the inventor of NMR (worth a Nobel prize) thought NMR would be used for "calibrating industrial magnets". Instead it was used to look inside people's brains. (But that was someone else's work, to apply it to that.)



                                                                                  On the other other hand, probably no politicians slapped Purcell & Bloch on the wrist when they said they were working on technology to calibrate industrial magnets. That can be envisaged by a politician and sounds science-y and maybe-important (just not as important as what it ended up being). Some mathematicians have also been known to name their ideas for marketing purposes, for example "dynamic programming" and "information entropy". (There are funny stories going along with the naming of each of these.)





                                                                                  If I had a piece of paper, my 10-second elevator pitch for algebraic topology would be:




                                                                                  1. Most data-analysis methods assume data is "Square" (meaning Euclidean--independent dimensions, flat, etc)

                                                                                  2. But how do we know that? What if the data is shaped like a peace sign? pax-benz

                                                                                  3. We don't want to be caught off-guard by assumptions we weren't necessarily trying to make, so do AT to be aware of other possibilities the world might hit you with.


                                                                                  thor vigfursson






                                                                                  share|cite|improve this answer











                                                                                  $endgroup$



                                                                                  I like the pictures from chapter 0 of Hatcher for that purpose.



                                                                                  hatcher chapter 0



                                                                                  And also Louis Kauffman made some nice pictures for the Fox calculus.



                                                                                  fox calculus



                                                                                  And Thurston's 3-manifolds book has good pictures as well.
                                                                                  thurston



                                                                                  "Normal maths works with squares, triangles, perfect circles; it counts beans. In topology we're freer to work with more interesting shapes." (cue picture from Dr Seuss)



                                                                                  seuss



                                                                                  This is basically what I did in http://tmblr.co/ZdCxIydt_Owi, drawing on the familiar philosophical distinction between reductivism vs holism to work in a cohomology element (as an argument for holism: some things can't be decomposed into atoms without losing some essential property that only shows up in how they're put together).



                                                                                  A romantic notion I felt when I was first reading Hatcher is that the "counting rocks" and "straight-sight cannonball" problems that seem more typical of a more boring mathematics, get replaced by "feminine" "weaving-based" mathematics. (Women were the ones tasked with purling, picking, and wrapping in most of history.) So this is a nice tie-in for non-mathematicians, although I'm not sure AT actually is more "feminine" (whatever that means) it's an attention-grabber.





                                                                                  You can also talk about topological spaces vs metric spaces by using highways



                                                                                  USA road network



                                                                                  versus airplane connections



                                                                                  paris air connection



                                                                                  taking a ring road versus trying to not back-track, versus taking small streets with different speed limits, could be thought of as metrics -- whereas if you're at the airport, then it's a question more of does the city you're in actually connect to where you're going or not, rather than 'physical' distance. You might end up flying past your destination (or at least extra distance) before taking a second flight/network hop, so the "network distance" is two (2 nbds), compare that to "ground distance". (Maybe for example Colorado Springs does not receive flights from L.A., so one has to go further east to Denver first ... if my geography is wrong then some pattern like what I'm suggesting does exist somewhere.)





                                                                                  As far as applications, I like the suggestions of Ghrist & Carlsson's work. I would add Ghrist's robots-not-bumping-into-each-other in warehouses, anyone can easily envisage as a braid the robots not bumping into each other over time. (See Ghrist's EAT book ch 1 for the relevant picture.)



                                                                                  ghrist braids robot motion planning factory floor



                                                                                  But I would make a further point, which is that the goal is not to rehash the applications already developed, but rather why the novel approaches might lead to something new. In other words, "How has this already been applied?" and "How might this generate applications in the future?" are two separate questions.



                                                                                  To that end I think it's fine, as long as you use clear non-technical language, to talk in generalities about how the "style" of topological thinking differs from maths they may be more familiar with. You could give some examples of "wiggly" or "non-rigid" situations and say that the ambition is that with enough development of AT, these situations could be handled.



                                                                                  Justin Curry made a pitch like this in http://arxiv.org/abs/1303.3255, saying essentially "Linear algebra was amazing for applications; I think my thing could be similarly productive, from a different starting point."



                                                                                  On the other hand, we probably all know examples of scientists who thought their research would be used for totally different purposes than what it ended up in. My favourite example is the inventor of NMR (worth a Nobel prize) thought NMR would be used for "calibrating industrial magnets". Instead it was used to look inside people's brains. (But that was someone else's work, to apply it to that.)



                                                                                  On the other other hand, probably no politicians slapped Purcell & Bloch on the wrist when they said they were working on technology to calibrate industrial magnets. That can be envisaged by a politician and sounds science-y and maybe-important (just not as important as what it ended up being). Some mathematicians have also been known to name their ideas for marketing purposes, for example "dynamic programming" and "information entropy". (There are funny stories going along with the naming of each of these.)





                                                                                  If I had a piece of paper, my 10-second elevator pitch for algebraic topology would be:




                                                                                  1. Most data-analysis methods assume data is "Square" (meaning Euclidean--independent dimensions, flat, etc)

                                                                                  2. But how do we know that? What if the data is shaped like a peace sign? pax-benz

                                                                                  3. We don't want to be caught off-guard by assumptions we weren't necessarily trying to make, so do AT to be aware of other possibilities the world might hit you with.


                                                                                  thor vigfursson







                                                                                  share|cite|improve this answer














                                                                                  share|cite|improve this answer



                                                                                  share|cite|improve this answer








                                                                                  edited Feb 10 at 22:08









                                                                                  Glorfindel

                                                                                  3,41381930




                                                                                  3,41381930










                                                                                  answered Aug 27 '14 at 2:12









                                                                                  isomorphismesisomorphismes

                                                                                  2,45912844




                                                                                  2,45912844























                                                                                      2












                                                                                      $begingroup$

                                                                                      The following examples come to mind. Show that the real projective plane, i.e., a Möbius strip glued with a disc along their boundaries, cannot be embedded in ${mathbb R}^3$. In its turn, you can additionally show that the Möbius strip (without boundary) is the space of all non-ordered pairs of points on a circle. There are many ways to prove these two statements, and some are quite elementary. I believe some proofs can be performed almost without words, just using pictures (well, elaborating pictures requires some serious work). You might take one and try to adapt it for "politicians". Good luck!






                                                                                      share|cite|improve this answer









                                                                                      $endgroup$


















                                                                                        2












                                                                                        $begingroup$

                                                                                        The following examples come to mind. Show that the real projective plane, i.e., a Möbius strip glued with a disc along their boundaries, cannot be embedded in ${mathbb R}^3$. In its turn, you can additionally show that the Möbius strip (without boundary) is the space of all non-ordered pairs of points on a circle. There are many ways to prove these two statements, and some are quite elementary. I believe some proofs can be performed almost without words, just using pictures (well, elaborating pictures requires some serious work). You might take one and try to adapt it for "politicians". Good luck!






                                                                                        share|cite|improve this answer









                                                                                        $endgroup$
















                                                                                          2












                                                                                          2








                                                                                          2





                                                                                          $begingroup$

                                                                                          The following examples come to mind. Show that the real projective plane, i.e., a Möbius strip glued with a disc along their boundaries, cannot be embedded in ${mathbb R}^3$. In its turn, you can additionally show that the Möbius strip (without boundary) is the space of all non-ordered pairs of points on a circle. There are many ways to prove these two statements, and some are quite elementary. I believe some proofs can be performed almost without words, just using pictures (well, elaborating pictures requires some serious work). You might take one and try to adapt it for "politicians". Good luck!






                                                                                          share|cite|improve this answer









                                                                                          $endgroup$



                                                                                          The following examples come to mind. Show that the real projective plane, i.e., a Möbius strip glued with a disc along their boundaries, cannot be embedded in ${mathbb R}^3$. In its turn, you can additionally show that the Möbius strip (without boundary) is the space of all non-ordered pairs of points on a circle. There are many ways to prove these two statements, and some are quite elementary. I believe some proofs can be performed almost without words, just using pictures (well, elaborating pictures requires some serious work). You might take one and try to adapt it for "politicians". Good luck!







                                                                                          share|cite|improve this answer












                                                                                          share|cite|improve this answer



                                                                                          share|cite|improve this answer










                                                                                          answered Oct 26 '13 at 18:10









                                                                                          Sasha Anan'inSasha Anan'in

                                                                                          20814




                                                                                          20814























                                                                                              1












                                                                                              $begingroup$

                                                                                              Roger Penrose in some of his lectures challenged students to find an everyday accessible example best described by second or higher cohomology.



                                                                                              I would construct an functional or relational description of a dynamic system and make some predictions about it using algebraic topology. Then you can use ordinary language. Try modelling two administrative systems using only relations of parts and show their topological equivalence, if it has to convince politicians...



                                                                                              I have some examples, but they are much too complicated to fit on one or two pages if stated verbally. And they might make you fail, depending on what politics your professor has, so be careful to choose examples which people don't have strong feelings about, if doing this.






                                                                                              share|cite|improve this answer









                                                                                              $endgroup$


















                                                                                                1












                                                                                                $begingroup$

                                                                                                Roger Penrose in some of his lectures challenged students to find an everyday accessible example best described by second or higher cohomology.



                                                                                                I would construct an functional or relational description of a dynamic system and make some predictions about it using algebraic topology. Then you can use ordinary language. Try modelling two administrative systems using only relations of parts and show their topological equivalence, if it has to convince politicians...



                                                                                                I have some examples, but they are much too complicated to fit on one or two pages if stated verbally. And they might make you fail, depending on what politics your professor has, so be careful to choose examples which people don't have strong feelings about, if doing this.






                                                                                                share|cite|improve this answer









                                                                                                $endgroup$
















                                                                                                  1












                                                                                                  1








                                                                                                  1





                                                                                                  $begingroup$

                                                                                                  Roger Penrose in some of his lectures challenged students to find an everyday accessible example best described by second or higher cohomology.



                                                                                                  I would construct an functional or relational description of a dynamic system and make some predictions about it using algebraic topology. Then you can use ordinary language. Try modelling two administrative systems using only relations of parts and show their topological equivalence, if it has to convince politicians...



                                                                                                  I have some examples, but they are much too complicated to fit on one or two pages if stated verbally. And they might make you fail, depending on what politics your professor has, so be careful to choose examples which people don't have strong feelings about, if doing this.






                                                                                                  share|cite|improve this answer









                                                                                                  $endgroup$



                                                                                                  Roger Penrose in some of his lectures challenged students to find an everyday accessible example best described by second or higher cohomology.



                                                                                                  I would construct an functional or relational description of a dynamic system and make some predictions about it using algebraic topology. Then you can use ordinary language. Try modelling two administrative systems using only relations of parts and show their topological equivalence, if it has to convince politicians...



                                                                                                  I have some examples, but they are much too complicated to fit on one or two pages if stated verbally. And they might make you fail, depending on what politics your professor has, so be careful to choose examples which people don't have strong feelings about, if doing this.







                                                                                                  share|cite|improve this answer












                                                                                                  share|cite|improve this answer



                                                                                                  share|cite|improve this answer










                                                                                                  answered Nov 4 '13 at 12:01







                                                                                                  user103756





































                                                                                                      draft saved

                                                                                                      draft discarded




















































                                                                                                      Thanks for contributing an answer to Mathematics Stack Exchange!


                                                                                                      • Please be sure to answer the question. Provide details and share your research!

                                                                                                      But avoid



                                                                                                      • Asking for help, clarification, or responding to other answers.

                                                                                                      • Making statements based on opinion; back them up with references or personal experience.


                                                                                                      Use MathJax to format equations. MathJax reference.


                                                                                                      To learn more, see our tips on writing great answers.




                                                                                                      draft saved


                                                                                                      draft discarded














                                                                                                      StackExchange.ready(
                                                                                                      function () {
                                                                                                      StackExchange.openid.initPostLogin('.new-post-login', 'https%3a%2f%2fmath.stackexchange.com%2fquestions%2f539675%2fhow-to-present-algebraic-topology-to-non-mathematicians%23new-answer', 'question_page');
                                                                                                      }
                                                                                                      );

                                                                                                      Post as a guest















                                                                                                      Required, but never shown





















































                                                                                                      Required, but never shown














                                                                                                      Required, but never shown












                                                                                                      Required, but never shown







                                                                                                      Required, but never shown

































                                                                                                      Required, but never shown














                                                                                                      Required, but never shown












                                                                                                      Required, but never shown







                                                                                                      Required, but never shown







                                                                                                      Popular posts from this blog

                                                                                                      Plaza Victoria

                                                                                                      In PowerPoint, is there a keyboard shortcut for bulleted / numbered list?

                                                                                                      How to put 3 figures in Latex with 2 figures side by side and 1 below these side by side images but in...